12.07.2015 Views

Chapter 1 Systems of Linear Equations and Matrices

Chapter 1 Systems of Linear Equations and Matrices

Chapter 1 Systems of Linear Equations and Matrices

SHOW MORE
SHOW LESS

Create successful ePaper yourself

Turn your PDF publications into a flip-book with our unique Google optimized e-Paper software.

<strong>Chapter</strong> 1<strong>Systems</strong> <strong>of</strong> <strong>Linear</strong> <strong>Equations</strong> <strong>and</strong><strong>Matrices</strong>System <strong>of</strong> linear algebraic equations <strong>and</strong> their solution constitute one <strong>of</strong> the major topicsstudied in the course known as “linear algebra.” In the first section we shall introducesome basic terminology <strong>and</strong> discuss a method for solving such system.1.1 <strong>Systems</strong> <strong>of</strong> <strong>Linear</strong> <strong>Equations</strong><strong>Linear</strong> <strong>Equations</strong>Any straight line in the xy-plane can be represented algebraically by an equation <strong>of</strong> theforma 1 x+a 2 y = bwhere a 1 , a 2 , <strong>and</strong> b are real constants <strong>and</strong> a 1 <strong>and</strong> a 2 are not both zero. An equation <strong>of</strong>this form is called a linear equation in the variables x <strong>and</strong> y. More generally, we definea linear equation in the n veriables x 1 ,x 2 ,...,x n to be one that can be expressed inthe forma 1 x 1 +a 2 x 2 +···+a n x n = bwhere a 1 ,a 2 ,...,a n , <strong>and</strong> b are real constants. The variables in a linear equation aresometimes called unknowns. For example, the equationsare linear. The equations3x+y = 7, y = 1 5 x+2z +4, <strong>and</strong> x 1 +3x 2 −2x 3 +5x 4 = 7√ x+3y = 2, 3x−2y −5z +yz = 4, <strong>and</strong> y = cosxare not linear.Observe that a linear equation does not involve any products or roots <strong>of</strong> variables. Allvariables occur only to the first power <strong>and</strong> do not appear as arguments for trigonometric,logarithmic, or exponential functions.1


Prepared by Dr.Archara Pacheenburawana (MA131 Sec 750001) 2A solution <strong>of</strong> a linear equation a 1 x 1 + a 2 x 2 + ··· + a n x n = b is a sequence <strong>of</strong> nnumbers s 1 , s 2 , ..., s n such that the equation satisfied when we substitute x 1 = s 1 ,x 2 = s 2 , ..., x n = s n . The set <strong>of</strong> all solutions <strong>of</strong> the equation is called its solution setor sometimes the general solution <strong>of</strong> the equation.Example 1.1 Find the solution set <strong>of</strong>(a) 5x−2y = 3 <strong>and</strong>(b) x 1 −3x 2 +4x 3 = 2.Solution .........<strong>Linear</strong> <strong>Systems</strong>A finite set <strong>of</strong> linear equations in the variables x 1 , x 2 , ..., x n is called system <strong>of</strong> linearequations or a linear system. A sequence <strong>of</strong> numbers s 1 , s 2 , ..., s n is called asolution <strong>of</strong> the system if x 1 = s 1 , x 2 = s 2 , ..., x n = s n is a solution <strong>of</strong> every equationin the system. For example, the system4x 1 −x 2 +3x 3 = −13x 1 +x 2 +9x 3 = −4the the solution x 1 = 1, x 2 = 2, <strong>and</strong> x 3 = −1 since these values satisfy both equations.However, x 1 = 1, x 2 = 8, x 3 = 1 is not a solution since these values satisfy only the firstequation in the system. Thus, not all system <strong>of</strong> linear equations have solutions.A system <strong>of</strong> equations that has no solutions is said to be inconsistent; if there isat least one solution <strong>of</strong> the system, it is called consistent. To illustrate the possibilitiesthat can occur in solving systems <strong>of</strong> linear equations, consider a general system <strong>of</strong> twolinear equations in the unknowns x <strong>and</strong> y:a 1 x+b 1 y = c 1a 2 x+b 2 y = c 2(a 1 ,b 1 not both zero)(a 2 ,b 2 not both zero)The graphs <strong>of</strong> these equations are lines; call them l 1 <strong>and</strong> l 2 . Since a point (x,y) lies ona line if <strong>and</strong> only if the number x <strong>and</strong> y satisfy the equation <strong>of</strong> the line, the solutions <strong>of</strong>the system <strong>of</strong> equations correspond to points <strong>of</strong> the intersection <strong>of</strong> l 1 <strong>and</strong> l 2 . There arethree possibilities, illustrated in Figure 1.1• The lines l 1 <strong>and</strong> l 2 may be parallel, in which case there is no intersection <strong>and</strong>consequently no solution to the system.• The lines l 1 <strong>and</strong> l 2 may intersect at only one point, in which case the system hasexactly one solution.• The lines l 1 <strong>and</strong> l 2 may coincide, in which case there are infinitely many points <strong>of</strong>intersection <strong>and</strong> consequently infinitely many solutions to the system.


Prepared by Dr.Archara Pacheenburawana (MA131 Sec 750001) 3yyyl 1 l 2l 1 l 2l 1 <strong>and</strong> l 2xxx(a) No solution(b) One solutionFigure 1.1:(c) Infinitely many solutionsAlthough we have considered only two equations with two unknowns here, we willshow later that the same three possibilities hold for arbitrary linear systems:Every system <strong>of</strong> linear equations has no solutions, or has exactly one solution,or has infinitely many solutions.An arbitrary system <strong>of</strong> m linear equations in n unknowns can be written asa 11 x 1 +a 12 x 2 +···+a 1n x n = b 1a 21 x 1 +a 22 x 2 +···+a 2n x n = b 2 (1.1)....a m1 x 1 +a m2 x 2 +···+a mn x n = b mwhere x 1 , x 2 , ..., x n are the unknowns <strong>and</strong> the subscripted a’s <strong>and</strong> b’s denote constants.For example, a general system <strong>of</strong> three linear equations in four unknowns can be writtenasa 11 x 1 +a 12 x 2 +a 13 x 3 +a 14 x 4 = b 1a 21 x 1 +a 22 x 2 +a 23 x 3 +a 24 x 4 = b 2a 31 x 1 +a 32 x 2 +a 33 x 3 +a 34 x 4 = b 3The double subscripting on the coefficients <strong>of</strong> the unknowns is a useful device thatis used to specify the location <strong>of</strong> the coefficient in the system. The first subscript onthe coefficient a ij indicates the equation in which the coefficient occurs, <strong>and</strong> the secondsubscript indicates which unknown it multiplies. Thus, a 12 is in the first equation <strong>and</strong>multiplies unknown x 2 .


Prepared by Dr.Archara Pacheenburawana (MA131 Sec 750001) 4Augmented <strong>Matrices</strong>A system <strong>of</strong> m linear equations in n unknowns can be abbreviated by writing only therectangular array <strong>of</strong> numbers⎡⎤a 11 a 12 ··· a 1n b 1a 21 a 22 ··· a 2n b 2⎢⎥⎣ . . . . ⎦ .a m1 a m2 ··· a mn b mThis is called the augmented matrix for the system. For example, the augmentedmatrix for the system <strong>of</strong> equationsisx 1 +x 2 +2x 3 = 92x 1 +4x 2 −3x 3 = 13x 1 +6x 2 −5x 3 = 0⎡⎣1 1 2 92 4 −3 13 6 −5 0Remark When constructing an augmented matrix, we must write the unknowns in thesame order in each equation, <strong>and</strong> the constants must be on the right.Exercise 1.11. Which <strong>of</strong> the following are linear equations in x 1 , x 2 <strong>and</strong> x 3 ?(a) x 1 +5x 2 − √ 2x 3 = 1 (b) x 1 +3x 2 +x 1 x 3 = 2(c) x 1 = −7x 2 +3x 3⎤⎦.(d) x −21 +x 2 +8x 3 = 5(e) x 3/51 −2x 2 +x 3 = 4 (f) πx 1 − √ 2x 2 + 1 3 x 3 = 7 1/32. Find the solution set <strong>of</strong> each <strong>of</strong> the following linear systems.(a) 7x−5y = 3 (b) 3x 1 −5x 2 +4x 3 = 7(c) −8x 1 +2x 2 −5x 3 +6x 4 = 1 (d) 3v −8w +2x−y +4z = 03. Which <strong>of</strong> the following are linear systems?(a) x 1 −3x 2 = x 3 −4x 4 = 1−x 1x 1 +x 4 +x 3 −2 = 0(c) 3x−xy = 1x+2xy −y = 0(e) 2x 1 −sinx 2 = 3x 2 = x 1 +x 3−x 1 +x 2 −3x 3 = 0(b) 2x− √ y +3z = −1x + 2y − z = 24x − y = −1(d) y = 2x−1y = −x(f) x 1 + x 2 +x 3 = 1−2x 2 1 − 2x 3 = −13x 2 −x 3 = 2


Prepared by Dr.Archara Pacheenburawana (MA131 Sec 750001) 54. Find the augmented matrix for each <strong>of</strong> the following systems <strong>of</strong> linear equations.(a) 3x 1 −2x 2 = −14x 1 +5x 2 = 37x 1 +3x 2 = 2(c) x 1 = 1x 2 = 2x 3 = 3(b) 2x 1 +2x 3 = 13x 1 −x 2 +4x 3 = 76x 1 +x 2 − x 3 = 0(d) x 1 + 2x 2 − x 4 + x 5 = 13x 2 + x 3 − x 5 = 2x 3 + 7x 4 = 15. Find a system <strong>of</strong> linear equations corresponding to the augmented matrix.⎡(a) ⎣(c)2 0 03 −4 00 1 1⎤⎦[ 7 2 1 −3 51 2 4 0 1]⎡(b) ⎣(d)⎡⎢⎣3 0 −2 57 1 4 −30 −2 1 71 0 0 0 70 1 0 0 −20 0 1 0 30 0 0 1 4⎤⎦⎤⎥⎦6. (a) Find a linear equation in the variables x <strong>and</strong> y that has the general solution(b) Show thatx = 5+2t, y = t.x = t, y = 1 2 t− 5 2is also the general solution <strong>of</strong> the equation in part (a).7. Consider the system <strong>of</strong> equationsax+by = kcx+dy = lex+fy = mIndicate what we can say about the relative positions <strong>of</strong> the line ax + by = k,cx+dy = l, <strong>and</strong> ex+fy = m when(a) the system has no solutions.(b) the system has exactly one solution.(c) the system has infinitely many solutions.8. Show that the linear systemax+by = ecx+dy = fwhere a,b,c,d,e,f are constants, has a unique solution if ad−bc ≠ 0. Express thissolution in terms <strong>of</strong> a,b,c,d,e, <strong>and</strong> f.


Prepared by Dr.Archara Pacheenburawana (MA131 Sec 750001) 61. (a), (c), (f)Answer to Exercise 1.12. (a) x = 3 7 + 5 7 t, y = t (b) x 1 = 5 3 s− 4 3 t− 1 8 , x 2 = s, x 3 = t(c) x 1 = 1 4 r − 5 8 s+ 3 4 t− 1 8 , x 2 = r, x 3 = s, x 4 = t(d) v = 8 q − 2 r + 1 s− 4 t, w = q, x = r, y = s, z = t3 3 3 33. (a), (d)⎡ ⎤3 −2 −1⎡2 0⎤2 1⎡4. (a) ⎣ 4 5 3 ⎦ (b) ⎣ 3 −1 4 7 ⎦ (c) ⎣7 3 2 6 1 −1 0⎡ ⎤1 2 0 −1 1 1(d) ⎣ 0 3 1 0 −1 2 ⎦0 0 1 7 0 15. (a) 2x 1 = 03x 1 −4x 2 = 0x 2 = 1(b) 3x 1 −2x 3 = 57x 1 +x 2 +4x 3 = −3−2x 2 +x 3 = 7(c) 7x 1 +2x 2 +x 3 −3x 4 = 5x 1 +2x 2 +4x 3 = 1(d) x 1 = 7x 2 = −2x 3 = 3x 4 = 41 0 0 10 1 0 20 0 1 36. (a) x−2y = 5 (b) Let x = t; then t−2y = 5. Solving for y yields y = 1 2 t− 5 2 .7. (a) The line have no common point <strong>of</strong> intersection.(b) The line intersect in exactly one point.(c) The three lines coincide.1.2 Elementary OperationsElementary Row OperationsThe basic method for solving a system <strong>of</strong> linear equations is to replace the given systemby a new system that has the same solution set but is easier to solve. This new system isgenerally obtained in a series <strong>of</strong> steps by applying the following three types <strong>of</strong> operationsto eliminate unknowns systematically:⎤⎦


Prepared by Dr.Archara Pacheenburawana (MA131 Sec 750001) 71. Multiply any equation, say the ith, by a nonzero constant α. This is denoted byαr i .2. Interchange <strong>of</strong> two equations, say the ith <strong>and</strong> jth. This is denoted by r i ↔ r j .3. Add α times the ith equation to the jth equation. This is denoted by r j + αr i .(This keeps the ith equation unchanged.)Since the rows <strong>of</strong> an augmented matrix correspond to the equations in the associatedsystem, these three operations correspond to the following operations on the rows <strong>of</strong> theaugmented matrix:1. Multiply any row by a nonzero constant: αr i .2. Interchange <strong>of</strong> two rows: r i ↔ r j .3. Add a multiple <strong>of</strong> one row to another: r j +αr i .These are called elementary row operations. The following illustrates how theseoperations can be used to solve system <strong>of</strong> linear equations.Example 1.2 Solve the following system by using elementary row operations.x+ y +2z = 92x+4y −3z = 13x+6y −5z = 0Solution In the left column we solve a system <strong>of</strong> linear equations by operating on theequations in the system, <strong>and</strong> in the right column we solve the same system by operatingon the rows <strong>of</strong> the augmented matrix.x+ y +2z = 92x+4y −3z = 13x+6y −5z = 0⎡⎣1 1 2 92 4 −3 13 6 −5 0⎤⎦Add −2 times the first equationto the second to obtainx+ y +2z = 92y −7z = −173x+6y −5z = 0Add −3 times the first equationto the third to obtainx+ y + 2z = 92y − 7z = −173y −11z = −27Multiply the second equation Multiplyby 1 to obtain2Add −2 times the first rowto the second to obtain⎡ ⎤1 1 2 9⎣ 0 2 −7 −17 ⎦3 6 −5 0Add −3 times the first rowto the third to obtain⎡ ⎤1 1 2 9⎣ 0 2 −7 −17 ⎦0 3 −11 −27the second rowby 1 to obtain2


Prepared by Dr.Archara Pacheenburawana (MA131 Sec 750001) 8x+ y + 2z = 9y − 7 2 z = −17 23y −11z = −27⎡⎣1 1 2 90 1 − 7 2− 17 20 3 −11 −27⎤⎦Add −3 times the second equationto the third to obtainx+ y +2z = 9y − 7 2 z = −17 2− 1 2 z = −3 2Multiply the third equationby −2 to obtainx+ y +2z = 9y − 7 2 z = −17 2z = 3Add −1 times the second equationto the first to obtainx + 11 2 z = 352y − 7 2 z = −17 2z = 3Add − 11 times the third equation2to the first <strong>and</strong> 7 times the third2equation to the second to obtainx = 1y = 2z = 3Add −3 times the second rowto the third to obtain⎡1 1 2 9⎤⎢⎣ 0 1 − 7 2− 17 ⎥2 ⎦0 0 − 1 2− 3 2Multiply the third rowby −2 to obtain⎡ ⎤1 1 2 9⎣ 0 1 − 7 − 17 ⎦2 20 0 1 3Add −1 times the second rowto the first to obtain⎡ ⎤11 351 02 2⎣ 0 1 − 7 − 17 ⎦2 20 0 1 3Add − 11 times the third row2to the first <strong>and</strong> 7 times the third2row to the second to obtain⎡ ⎤1 0 0 1⎣ 0 1 0 2 ⎦0 0 1 3The solution x = 1, y = 2, z = 3 is now evident. ♣Echelon FormsIn previous example, we solved a linear system in the unknowns x, y, <strong>and</strong> z by reducingthe augmented matrix to the form⎡ ⎤1 0 0 1⎣ 0 1 0 2 ⎦.0 0 1 3This is an example <strong>of</strong> a matrix that is in reduced row-echelon form. To be <strong>of</strong> thisform, a matrix must have the following properties:1. If the row does not consist entirely <strong>of</strong> zeros, then the first nonzero number in therow is a 1. We call this a leading 1.


Prepared by Dr.Archara Pacheenburawana (MA131 Sec 750001) 92. If there are any rows that consist entirely <strong>of</strong> zeros, then they are grouped togetherat the bottom <strong>of</strong> the matrix.3. In any two successive rows that do not consist entirely <strong>of</strong> zeros, the leading 1 inthe lower row occurs farther to the right than the leading 1 in the higher row.4. Each column that contains a leading 1 has zeros everywhere else in that column.A matrix that has the first three properties is said to be in row-echelon matrix.(Thus, a matrix in reduced row-echelon form is <strong>of</strong> necessity in row-echelon form, but notconversely.)The following matrices are in reduced row-echelon form.⎡ ⎤⎡ ⎤ ⎡ ⎤ 0 1 −2 0 01 0 0 4 1 0 0[ ]⎣0 1 0 7 ⎦, ⎣0 1 0⎦,⎢0 0 0 1 3⎥ 0 0⎣0 0 0 0 0⎦ , 0 00 0 1 −1 0 0 10 0 0 0 0The following matrices are in row-echelon form.⎡ ⎤ ⎡ ⎤1 4 −3 7 1 0 0⎣0 1 6 2 ⎦, ⎣0 1 0⎦,0 0 1 −1 0 0 0⎡ ⎤0 1 2 6 0⎣0 0 1 −1 0⎦0 0 0 0 1Example 1.3 Suppose that the augmented matrix for a linear equations has been reducedby row operations to the given reduced row-echelon form. Solve the system.⎡ ⎤ ⎡ ⎤1 0 0 21 0 0 3 −5(a) ⎣ 0 1 0 4 ⎦ (b) ⎣ 0 1 0 4 −1 ⎦0 0 1 −30 0 1 2 6⎡(c) ⎣1 0 2 −30 1 1 40 0 0 0Solution .........Elimination Methods⎤⎦⎡(d) ⎣1 0 4 00 1 −5 00 0 0 1We have just seen how easy it is to solve a system <strong>of</strong> linear equations once its augmentedmatrix is in reduced row-echelon form.• The process <strong>of</strong> using row operations to transform a linear system into one whoseaugmented matrix is in row-echelon form is called Gaussian elimination.• The process <strong>of</strong> using row operations to transform a linear system into one whoseaugmented matrix is in reduced row-echelon form is called Gauss-Jordan elimination.⎤⎦


Prepared by Dr.Archara Pacheenburawana (MA131 Sec 750001) 10Remark It can be show that every matrix has a unique reduced row-echelon form.Example 1.4 Solve by Gauss-Jordan elimination.x 1 +3x 2 −2x 3 +2x 5 = 02x 1 +6x 2 −5x 3 −2x 4 +4x 5 −3x 6 = −15x 3 +10x 4 +15x 6 = 52x 1 +6x 2 +8x 4 +4x 5 +18x 6 = 6Solution .........Back-SubstitutionIt is sometimes preferable to solve a system <strong>of</strong> linear equations by using Gaussian eliminationto bring the augmented matrix into row-echelon form without continuing all theway to the reduced row-echelon form. When this is done, the corresponding system <strong>of</strong>equations can be solved by a technique called back-substitution. The next exampleillustrates the idea.Example 1.5 From Example 1.4, solve by Gaussian elimination <strong>and</strong> back-substitution.Solution .........Example 1.6 Solvex+y +2z = 92x+4y −3z = 13x+6y −5z = 0by Gaussian elimination <strong>and</strong> back-substitution.Solution .........Homogeneous <strong>Linear</strong> <strong>Systems</strong>A system <strong>of</strong> linear equations is said to be homogeneous if the constant terms are allzero; that is the system has the forma 11 x 1 +a 12 x 2 +···+a 1n x n = 0a 21 x 1 +a 22 x 2 +···+a 2n x n = 0..a m1 x 1 +a m2 x 2 +···+a mn x n = 0Every homogeneous system <strong>of</strong> linear equations is consistent, since all such systemshave x 1 = 0, x 2 = 0, ..., x n = 0 as a solution. This solution is called the trivialsolution; if there are other solutions, they are called nontrivial solution.Because a homogeneous linear system always has the trivial solution, there are onlytwo possibilities for its solutions:..


Prepared by Dr.Archara Pacheenburawana (MA131 Sec 750001) 11• The system has only the trivial solution.• The system has infinitely many solutions in addition to the trivial solution.In the special case <strong>of</strong> a homogeneous linear system <strong>of</strong> two equations in two unknowns,saya 1 x+b 1 y = 0 (a 1 ,b 1 not both zero)a 2 x+b 2 y = 0 (a 2 ,b 2 not both zero)the graphs <strong>of</strong> the equations are lines through the origin, <strong>and</strong> the trivial solution correspondsto the point <strong>of</strong> intersection at the origin.yya 1 x+b 1 y = 0xa 2 x+b 2 y = 0(a) Only the trivial solutionxa 1 x+b 1 y = 0<strong>and</strong>a 2 x+b 2 y = 0(b) Infinitely many solutionsThere is one case in which a homogeneous system is assured <strong>of</strong> having nontrivialsolution — namely, whenever the system involves more unknowns than equations. Tosee why, consider the following example <strong>of</strong> four equations in five unknowns.Example 1.7 Solve the following homogeneous system <strong>of</strong> linear equations by usingGauss-Jordan elimination.Solution .........2x 1 +2x 2 −x 3 +x 5 = 0−x 1 −x 2 +2x 3 −3x 4 +x 5 = 0x 1 +x 2 −2x 3 −x 5 = 0x 3 +x 4 +x 5 = 0Theorem 1.1 A homogeneous system <strong>of</strong> linear equations with more unknowns thanequations has infinitely many solutions.Remark Note that Theorem 1.1 applied only to homogeneous systems. A nonhomogeneoussystem with more unknowns than equations need not be consistent; however, ifthe system is consistent, it will have infinitely many solutions.


Prepared by Dr.Archara Pacheenburawana (MA131 Sec 750001) 12Exercise 1.21. Which <strong>of</strong> the following 3×3 matrices are in reduced row-echelon form?⎡ ⎤1 0 0(a) ⎣0 1 0⎦0 0 1⎡ ⎤1 0 0(e) ⎣0 0 0⎦0 0 1⎡ ⎤0 1 0(b) ⎣1 0 0⎦0 0 0⎡ ⎤1 1 0(f) ⎣0 1 0⎦0 0 0⎡ ⎤0 1 0(c) ⎣0 0 1⎦0 0 0⎡ ⎤1 0 2(g) ⎣0 1 3⎦0 0 0⎡ ⎤1 0 0(d) ⎣0 0 1⎦0 0 0⎡ ⎤0 0 0(h) ⎣0 0 0⎦0 0 02. Which <strong>of</strong> the following 3×3 matrices are in row-echelon form?⎡ ⎤1 0 0(a) ⎣0 1 0⎦0 0 1⎡ ⎤1 4 6(e) ⎣0 0 1⎦0 1 3⎡ ⎤1 3 0(b) ⎣0 0 1⎦0 0 0⎡ ⎤1 1 0(f) ⎣0 1 0⎦0 0 0⎡ ⎤1 1 1(c) ⎣0 1 2⎦0 0 3⎡ ⎤1 3 4(g) ⎣0 0 1⎦0 0 0⎡ ⎤1 0 0(d) ⎣0 1 0⎦0 2 0⎡ ⎤1 2 3(h) ⎣0 0 0⎦0 0 13. In each part determine whether the matrix is in row-echelon form, reduced rowechelonform, both, or neither.⎡ ⎤ ⎡ ⎤0 1 3 5 7 1 0 0 5 [ ](a) ⎣0 0 1 2 3⎦(b) ⎣0 0 1 3⎦1 0 3 1(c)0 1 2 40 0 0 0 0 0 1 0 4(d)[ ]1 −7 5 50 1 3 2⎡ ⎤1 0 0 1 2(e) ⎣0 1 0 2 4⎦0 0 1 3 6⎡ ⎤0 1(f) ⎣0 0⎦0 04. Find the reduced row-echelon form <strong>of</strong> A when⎡⎤2 2 −1 0 1 0A = ⎢−1 −1 2 −3 1 0⎥⎣ 1 1 −2 0 −1 0⎦ .0 0 1 1 1 05. Find the reduced row-echelon form <strong>of</strong> B when⎡ ⎤0 0 −2 0 7 12B = ⎣2 4 −10 6 12 28⎦.2 4 −5 6 −5 −16. In each part suppose that the augmented matrix for a system <strong>of</strong> linear equationshas been reduced by row operations to the given reduced row-echelon form. Solvethe system.


Prepared by Dr.Archara Pacheenburawana (MA131 Sec 750001) 13⎡(a) ⎣(c) ⎣(e)⎡⎡⎢⎣1 0 0 −30 1 0 00 0 1 71 −3 0 20 0 1 −20 0 0 0⎤⎦⎤⎦1 −6 0 0 3 −20 0 1 0 4 70 0 0 1 5 80 0 0 0 0 0⎤⎥⎦⎡(b) ⎣(d)1 0 0 −7 80 1 0 3 20 0 1 1 −5[ 1 2 0 1 50 0 1 3 4⎡(f) ⎣1 −3 0 00 0 1 00 0 0 1]⎤⎦⎤⎦7. Solve each <strong>of</strong> the following systems by Gauss-Jordan elimination.(a) x 1 + x 2 +2x 3 = 8−x 1 −2x 2 +3x 3 = 13x 1 −7x 2 +4x 3 = 10(c) 2x 1 −3x 2 = −22x 1 + x 2 = 13x 1 +2x 2 = 1(e) 4x 1 −8x 2 = 123x 1 −6x 2 = 9−2x 1 +4x 2 = −6(b) 2x 1 +2x 2 +2x 3 = 0−2x 1 +5x 2 +2x 3 = 18x 1 + x 2 +4x 3 = −1(d) −2b+3c = 13a+6b−3c = −26a+6b+3c = 5(f) x 1 +3x 2 + x 3 + x 4 = 32x 1 −2x 2 + x 3 +2x 4 = 83x 1 + x 2 +2x 3 − x 4 = −1(g)x− y +2z − w = −12x+ y −2z −2w = −2−x+2y −4z + w = 13x −3w = −3(h)3x 1 +2x 2 − x 3 = −155x 1 +3x 2 +2x 3 = 03x 1 + x 2 +3x 3 = 11−6x 1 −4x 2 +2x 3 = 30(i) x 1 + x 2 +x 3 + x 4 = 02x 1 + x 2 −x 3 +3x 4 = 0x 1 −2x 2 +x 3 + x 4 = 0(j) 10y −4z + w = 1x+ 4y − z + w = 23x+2y + z +2w = 5−2x−8y +2z −2w = −4x−6y +3z = 18. Solve each <strong>of</strong> the systems in Exercise 2 by Gaussian elimination.9. Determine which <strong>of</strong> the following homogeneous systems have nontrivial solutions.(a) 2x 1 −3x 2 +4x 3 − x 4 = 07x 1 + x 2 −8x 3 +9x 4 = 02x 1 +8x 2 + x 3 − x 4 = 0(c) a 11 x 1 +a 12 x 2 +a 13 x 3 = 0a 21 x 1 +a 22 x 2 +a 23 x 3 = 0(b) x 1 +3x 2 − x 3 = 0x 2 −8x 3 = 04x 3 = 0(d) 3x 1 −2x 2 = 06x 1 −4x 2 = 0


Prepared by Dr.Archara Pacheenburawana (MA131 Sec 750001) 1410. Solve the following homogeneous systems <strong>of</strong> linear equations by any method.(a) 2x 1 +x 2 +3x 3 = 0x 1 +2x 2 = 0x 2 + x 3 = 0(c) −x 1 +x 2 − x 3 +3x 4 = 03x 1 +x 2 − x 3 − x 4 = 02x 1 −x 2 −2x 3 − x 4 = 0(e) 2x+2y +4z = 0w − y −3z = 02w+3x+ y + z = 0−2w + x+3y −2z = 0(b) 3x 1 +x 2 +x 3 +x 4 = 05x 1 −x 2 +x 3 −x 4 = 0(d) − 1 x 2 1 + 1 x 3 2 + 1 x 2 3 = 01x 4 1 − 2x 3 2 + 1x 4 3 = 01x 4 1 + 1x 3 2 − 3x 4 3 = 0(f) 2x− y −3z = 0−x+2y −3z = 0x+ y +4z = 011. Consider a linear system whose augmented matrix is <strong>of</strong> the form⎡ ⎤1 2 1 1⎣ −1 4 3 2 ⎦2 −2 α 3For what values <strong>of</strong> α will the system have a unique solution?12. Consider a linear system whose augmented matrix is <strong>of</strong> the form⎡ ⎤1 2 1 0⎣ 2 5 3 0 ⎦−1 1 β 0(a) Is it possible for the system to be consistent? Explain.(b) For what values <strong>of</strong> β will the system have infinitely many solutions?13. Consider a linear system whose augmented matrix is <strong>of</strong> the form⎡ ⎤1 1 3 2⎣ 1 2 4 3 ⎦1 3 α β(a) For what values <strong>of</strong> α <strong>and</strong> β will the system have infinitely many solutions?(b) For what values <strong>of</strong> α <strong>and</strong> β will the system be inconsistent?14. Solve the system2x 1 −x 2 = λx 12x 1 −x 2 + x 3 = λx 2−2x 1 +2x 2 +x 3 = λx 3for x 1 , x 2 , <strong>and</strong> x 3 in the two cases λ = 1, λ = 2.


Prepared by Dr.Archara Pacheenburawana (MA131 Sec 750001) 15Answer to Exercise 1.21. (a), (b), (d), (g), (h) 2. (a), (b), (c), (f), (g)3. (a) Row-echelon (b) Neither (c) Both (d) Row-echelon (e) Both (f) Both⎡ ⎤1 1 0 0 1 0 ⎡ ⎤4. ⎢0 0 1 0 1 01 2 0 3 0 7⎥⎣0 0 0 1 0 0⎦ 5. ⎣0 0 1 0 0 1⎦0 0 0 0 1 20 0 0 0 0 06. (a) x 1 = 3, x 2 = 0, x 3 = 7 (b) x 1 = 7t+8, x 2 = −3t+2, x 3 = −t−5, x 4 = t(c) x 1 = 2+3t, x 2 = t, x 3 = −2 (d) x 1 = 5−2s−t, x 2 = s, x 3 = 4−3t, x 4 = t(e) x 1 = 6s−3t−2, x 2 = s, x 3 = −4t+7, x 4 = −5t+8, x 5 = t(f) Inconsistent7. (a) x 1 = 3, x 2 = 1, x 3 = 2 (b) x 1 = − 1 7 − 3 7 t, x 2 = 1 7 − 4 7 t, x 3 = t(c) Inconsistent (d) Inconsistent (e) x 1 = 3+2t, x 2 = t(f) x 1 = 3 4 − 5 8 t, x 2 = − 1 4 − 1 8 t, x 3 = t, x 4 = 3(g) x = t−1, y = 2s, z = s, w = t (h) x 1 = −4, x 2 = 2, x 3 = 7(i) x 1 = − 4 3 t, x 2 = 0, x 3 = 1 3 t, x 4 = t(j) x = 14 − 6 t, y = − 7 + 3 t− 1 s, z = −2+t, w = t5 5 10 10 109. (a), (c), (d)10. (a) x 1 = 0, x 2 = 0, x 3 = 0(b) x 1 = −s, x 2 = −t−s, x 3 = 4s, x 4 = t(c) x 1 = t, x 2 = −t, x 3 = t (d) x 1 = 5 3 t, x 2 = t, x 3 = t(e) w = t, x = −t, y = t, z = 0 (f) x = 0, y = 0, z = 011. α ≠ −2 12. β = 2 13. (a) α = 5, β = 4, (b) α = 5, β ≠ 414. If λ = 1, then x 1 = x 2 = s, x 3 = 0. If λ = 2, then x 1 = − 1 2 s, x 2 = 0, x 3 = s1.3 <strong>Matrices</strong> <strong>and</strong> Matrix OperationsIn this section we begin our study <strong>of</strong> matrix theory by giving some <strong>of</strong> the fundamentaldefinitions <strong>of</strong> the subject. We shall see how matrices can be combined through thearithmetic operations <strong>of</strong> addition, subtraction, <strong>and</strong> multiplication.


Prepared by Dr.Archara Pacheenburawana (MA131 Sec 750001) 16Matrix Notation <strong>and</strong> TerminologyDefinition 1.1 A matrix is a rectangular array <strong>of</strong> numbers, each <strong>of</strong> which is called anentry <strong>of</strong> the matrix.For example,⎡ ⎤2 1⎣−1 2⎦ (1.2)0 5( )1 −1 0(1.3)0 5 0<strong>and</strong> [1√2 π](1.4)Each horizontal line <strong>of</strong> number is called a row <strong>of</strong> the matrix; each vertical one iscalled a column. For example, in matrix (1.2) the third row consist <strong>of</strong> the entries 0 <strong>and</strong>5, whereas the entries in the first column are 2, −1, <strong>and</strong> 0.If the given matrix has m rows <strong>and</strong> n columns, it is called m×n matrix (read, mby n matrix), <strong>and</strong> the numbers m <strong>and</strong> n are the sizes <strong>of</strong> the matrix. The matrices (1.2),(1.3), <strong>and</strong> (1.4), are respectively, 3×2, 2×3, <strong>and</strong> 1×3 matrices.A matrix with only one column or m ×1 matrix (with m > 1) is called a columnmatrix (or a column vector), <strong>and</strong> a matrix with only one row or 1×n matrix (withn > 1) is called a row matrix (or a row vector). For example, the matrix (1.4) iscalled a row matrix or a row vector.Capital (uppercase) letters, such as A, B, <strong>and</strong> C, will be used to denote matrices,whereas the entries <strong>of</strong> a matrix will be denoted by the corresponding lowercase letterwith “double subscripts.” The entry that occurs in row i <strong>and</strong> column j <strong>of</strong> a matrix Awill be denoted by a ij . Thus, a general 2×4 matrix might be written as[ ]a11 aA = 12 a 13 a 14a 21 a 22 a 23 a 24<strong>and</strong> a general m×n matrix as⎡ ⎤a 11 a 12 ··· a 1na 21 a 22 ··· a 2nA = ⎢ ⎥⎣ . . . ⎦ .a m1 a m2 ··· a mnWhen compactness <strong>of</strong> notation is desired, the preceding matrix can be written as[a ij ] m×n <strong>and</strong> [a ij ]the first notation being used when it is important in the discussion to know the size, <strong>and</strong>the second being used when the size need not be emphasized.


Prepared by Dr.Archara Pacheenburawana (MA131 Sec 750001) 17The entry in row i <strong>and</strong> column j <strong>of</strong> a matrix A is also commonly denoted by thesymbol (A) ij . Thus, for the matrix⎡ ⎤3 6 −1A = ⎣4 2 0 ⎦1 −3 −5we have (A) 12 = a 12 = 6, (A) 21 = a 21 = 4, <strong>and</strong> (A) 33 = a 33 = 5.Row <strong>and</strong> column matrices are <strong>of</strong> special importance, <strong>and</strong> it is common practice todenote them by boldface lowercase letters rather than capital letters. For such matrices,double subscripting <strong>of</strong> the entries is unnecessary. Thus a general 1×n row matrix a <strong>and</strong>a general m×1 column matrix b would be written as⎡ ⎤b 1a = [ ]b 2a 1 a 2 ··· a n <strong>and</strong> b = ⎢ ⎥⎣ . ⎦b nA matrix A with n rows <strong>and</strong> n columns is called a square matrix <strong>of</strong> order n, <strong>and</strong>the entries a 11 , a 22 , ..., a nn in (1.5) are said to be on the main diagonal <strong>of</strong> A.⎡ ⎤a 11 a 12 ··· a 1na 21 a 22 ··· a 2n⎢ ⎥⎣ . . . ⎦ . (1.5)a n1 a n2 ··· a nnDefinition 1.2 A square matrix in which all the entries above the main diagonal arezero is called lower triangular, <strong>and</strong> a square matrix in which all the entries belowthe main diagonal are zero is called upper triangular. A matrix that is either uppertriangular or lower triangular is called triangular.Operations on <strong>Matrices</strong>Definition 1.3 Given matrices A <strong>and</strong> B, the entries a ij <strong>and</strong> b kp are correspondingentries if <strong>and</strong> only if i = k <strong>and</strong> j = p. In other words, an entry <strong>of</strong> A corresponds toan entry <strong>of</strong> B if <strong>and</strong> only if they occupy the same position in their respective matrices.For example, ifA =[ ] 1 2 34 5 6<strong>and</strong> B =[ ] 7 8 9−1 −2 −3then 1 <strong>and</strong> 7, 2 <strong>and</strong> 8, <strong>and</strong> 4 <strong>and</strong> −1 are all pairs <strong>of</strong> corresponding entries. We shallspeak <strong>of</strong> corresponding entries only when the two matrices in question have the samesize.Definition 1.4 Equality <strong>of</strong> matrices. Two matrices A <strong>and</strong> B are equal, writtenA = B, if they have the same size <strong>and</strong> their corresponding entries are equal.In matrix notation, if A = [a ij ] <strong>and</strong> B = [b ij ] have the same size, then A = B if <strong>and</strong>only if a ij = b ij for all i <strong>and</strong> j.


Prepared by Dr.Archara Pacheenburawana (MA131 Sec 750001) 18Example 1.8 Which <strong>of</strong> the following matrices are equal:[ ] [ ]1 2 −1 1 2A = , B = ,4 0 1+2 4 0Solution .........[ ] 14−1C = 2 , D =4 0 3[ ] 1 2 −1?4 0 1Definition 1.5 Sum <strong>of</strong> matrices. Let A <strong>and</strong> B be matrices with the same size. Thesum <strong>of</strong> A <strong>and</strong> B, written A+B, is the matrix obtained by adding corresponding entries<strong>of</strong> A <strong>and</strong> BIn matrix notation, if A = [a ij ] <strong>and</strong> B = [b ij ] have the same size, then C = A+B if<strong>and</strong> only if c ij = a ij +b ij for all i <strong>and</strong> j.[ ] 2 −1 5Example 1.9 Let A = , B =3 0 −4A+B <strong>and</strong> A+C, if definedSolution .........[ ] −3 2 7, <strong>and</strong> C =5 −6 2[ ] 3 5. Find−2 −1Definition 1.6 Multiplication by a scalar. Let A be a matrix <strong>and</strong> c be a scalar.Then the product cA is the matrix obtained by multiplying each entry <strong>of</strong> A by scalar c.The matrix cA is said to be a scalar multiple <strong>of</strong> A.In matrix notation, if A = [a ij ], then B = cA if <strong>and</strong> only if b ij = ca ij for all i <strong>and</strong> j.Definition 1.7 The negative <strong>of</strong> a matrix B, written −B, is the matrix (−1)B. It isobtained from B by simply reversing the sign <strong>of</strong> every entry.Example 1.10 LetFind 3A <strong>and</strong> (−1)A.A =[ ]1 −1 0 3.2 6 −4 8Solution .........Definition 1.8 Difference <strong>of</strong> matrices. Let A <strong>and</strong> B be matrices with the same size.The difference <strong>of</strong> A <strong>and</strong> B, written A−B, is the matrix C given by C = A+(−B).Note The matrix A−B can be obtained by simply subtracting each entry <strong>of</strong> B fromthe corresponding entry <strong>of</strong> A.⎡ ⎤ ⎡ ⎤5 4 −3 0Example 1.11 Find C = 2A−B where A = ⎣−1 7 ⎦ <strong>and</strong> B = ⎣ 4 2 ⎦.9 −3 5 −7


Prepared by Dr.Archara Pacheenburawana (MA131 Sec 750001) 19Solution .........If A 1 ,A 2 ,...,A n are matrices <strong>of</strong> the same size <strong>and</strong> c 1 ,c 2 ,...,c n are scalars, then anexpression <strong>of</strong> the formc 1 A 1 +c 2 A 2 +···+c n A nis called a linear combination <strong>of</strong> A 1 ,A 2 ,...,A n with coefficients c 1 , c 2 , ..., c n . Forexample, If[ ] [ ] [ ]2 3 4 0 2 7 9 −6 3A = , B = , <strong>and</strong> C = ,1 3 1 −1 3 −5 3 0 12then2A−B + 1 [ ] 2 3 43 C = 2 1 3 1[ ] 4 6 8= +2 6 2[ ] 7 2 2=4 3 11[ 0 2 7+(−1)−1 3 −5][ 0 −2 −71 −3 5+]+ 1 [ ] 9 −6 33 3 0 12[ ] 3 −2 11 0 4is the linear combination <strong>of</strong> A, B, <strong>and</strong> C with scalar coefficients 2, −1, <strong>and</strong> 1 3 .Definition 1.9 Matrix multiplication. If A is an m×r matrix <strong>and</strong> B is an r ×nmatrix, then the product AB is the m × n matrix whose entries are determined asfollows. To find the entry in row i <strong>and</strong> column j <strong>of</strong> AB, single out row i from matrixA <strong>and</strong> column j from matrix B. Multiply the corresponding entries from the row <strong>and</strong>column together, <strong>and</strong> then add up the resulting products.In other words, if C = AB, thenor, using summation notation,c ij = a i1 b 1j +a i2 b 2j +···+a in b njc ij =n∑a ik b kj .k=1Example 1.12 Given the pairs <strong>of</strong> matrices A <strong>and</strong> B, find the products AB <strong>and</strong> BA, ifdefined.⎡ ⎤[ ] 4 1 4 31 2 4(a) A = , B = ⎣0 −1 3 1⎦2 6 02 7 5 2⎡ ⎤ ⎡ ⎤1 2 3 x(b) A = ⎣4 5 6⎦, B = ⎣y⎦7 8 9 zSolution .........Note It should be apparent from Example 1.12 that multiplication <strong>of</strong> matrices “actdifferently” frommultiplication<strong>of</strong>numbers. Thelatterhastheproperty<strong>of</strong>commutativity(for all numbers a <strong>and</strong> b, ab = ba), but matrix multiplication does not.One situation in which the products AB <strong>and</strong> BA are both defined (although theystill need not be equal) is when A <strong>and</strong> B have the same size <strong>and</strong> are square.


Prepared by Dr.Archara Pacheenburawana (MA131 Sec 750001) 20Partitioned <strong>Matrices</strong>A matrix can be subdivided or partitioned into small matrices by inserting horizontal<strong>and</strong> vertical rules between selected rows <strong>and</strong> columns. For example, the following arethree possible partitions <strong>of</strong> a general 3×4 matrix A:• partition <strong>of</strong> A into four submatrices A 11 , A 12 , A 21 , <strong>and</strong> A 22⎡ ⎤a 11 a 12 a 13 a 14[ ]A = ⎣ a 21 a 22 a 23 a 24⎦ A11 A= 12Aa 31 a 32 a 33 a 21 A 2234• partition <strong>of</strong> A into its row matrices r 1 , r 2 , <strong>and</strong> r 3⎡ ⎤ ⎡ ⎤a 11 a 12 a 13 a 14 r 1A = ⎣ a 21 a 22 a 23 a 24⎦ = ⎣r 2⎦a 31 a 32 a 33 a 34 r 3• partition <strong>of</strong> A into its column matrices c 1 , c 2 , c 3 , <strong>and</strong> c 4⎡ ⎤a 11 a 12 a 13 a 14A = ⎣ a 21 a 22 a 23 a 24⎦ = [ ]c 1 c 2 c 3 c 4a 31 a 32 a 33 a 34Matrix Multiplication by Columns <strong>and</strong> by RowsSometimes it may bedesirable t<strong>of</strong>ind aparticular rowor column<strong>of</strong> a matrixproduct ABwithout computing the entire product. The following results are useful for that purpose:ith row matrix <strong>of</strong> AB = [ ith row matrix <strong>of</strong> A ] B (1.6)jth column <strong>of</strong> AB = A [ jth column <strong>of</strong> B ] (1.7)Example 1.13 If A <strong>and</strong> B are the matrices in Example 1.12(a), then find the first row<strong>and</strong> second column <strong>of</strong> ABSolution .........If a 1 , a 2 , ..., a m denote the row matrices <strong>of</strong> A <strong>and</strong> b 1 , b 2 , ..., b n denote the columnmatrices <strong>of</strong> B, then it follows from Formula (1.6) <strong>and</strong> (1.7) that⎡ ⎤ ⎡ ⎤a 1 a 1 Ba 2AB = ⎢ ⎥⎣ . ⎦ B = a 2 B⎢ ⎥(1.8)⎣ . ⎦a m a m B(AB computed row by row)AB = A [ b 1 b 2 ··· b n]=[Ab1 Ab 2 ··· Ab n](1.9)(AB computed column by column)


Prepared by Dr.Archara Pacheenburawana (MA131 Sec 750001) 21Matrix Products as <strong>Linear</strong> CombinationsSuppose thatThen⎡ ⎤ ⎡ ⎤a 11 a 12 ··· a 1n x 1a 21 a 22 ··· a 2nA = ⎢ ⎥⎣ . . . ⎦ <strong>and</strong> x = x 2⎢ ⎥⎣ . ⎦a m1 a m2 ··· a mn x n⎡⎤a 11 x 1 +a 12 x 2 +···+a 1n x na 21 x 1 +a 22 x 2 +···+a 2n x nAx = ⎢⎥⎣ . . . ⎦a m1 x 1 +a m2 x 2 +···+a mn x n⎡ ⎤ ⎡ ⎤ ⎡ ⎤a 11 a 12 a 1na 21= x 1 ⎢ ⎥⎣ . ⎦ +x a 222⎢⎥⎣ . ⎦ +···+x a 2nn⎢⎥⎣ . ⎦a m1 a m2 a mn(1.10)In words, (1.10) tell us that the product Ax <strong>of</strong> the matrix A with a column matrix xis a linear combination <strong>of</strong> the column matrices <strong>of</strong> A with the coefficients coming fromthe matrix x. Moreover, the product yA <strong>of</strong> a 1×m matrix y with an m×n matrix A isa linear combination <strong>of</strong> the row matrices <strong>of</strong> A with scalar coefficients coming from y.Example 1.14 The matrix product⎡ ⎤⎡⎤ ⎡ ⎤2 −1 3 −1 5⎣ 1 3 −2⎦⎣2 ⎦ = ⎣−1⎦−3 2 1 3 10can be written as the linear combination <strong>of</strong> the column matrices⎡ ⎤ ⎡ ⎤ ⎡ ⎤ ⎡ ⎤2 −1 3 5−1⎣1 ⎦+2⎣3 ⎦+3⎣−2⎦ = ⎣−1⎦−3 2 1 10The matrix product⎡ ⎤[ ]2 −1 35 −1 10 ⎣ 1 3 −2⎦ = [ −21 12 27 ]−3 2 1can be written as the linear combination <strong>of</strong> the row matrices5 [ 2 −1 3 ] −1 [ 1 3 −2 ] +10 [ −3 2 1 ] = [ −21 12 27 ] ♣It follows from (1.8) <strong>and</strong> (1.10) that the jth column matrix <strong>of</strong> a product AB is alinear combination <strong>of</strong> the column matrices <strong>of</strong> A with the coefficients coming from the jthcolumn <strong>of</strong> B.


Prepared by Dr.Archara Pacheenburawana (MA131 Sec 750001) 22Example 1.15 We showed in Example 1.12(a) that⎡ ⎤ ⎡ ⎤3 −2 [ ] −14 1 −3AB = ⎣2 4 ⎦ −2 1 3= ⎣ 12 6 30 ⎦4 1 61 −3 −14 −2 −15The column matrices <strong>of</strong> AB can be expressed as linear combinations <strong>of</strong> the column matrices<strong>of</strong> A as follows:⎡ ⎤ ⎡ ⎤ ⎡ ⎤−14 3 −2⎣ 12 ⎦ = −2⎣2⎦+4⎣4 ⎦−14 1 −3⎡ ⎤ ⎡ ⎤ ⎡ ⎤1 3 −2⎣ 6 ⎦ = 1⎣2⎦+1⎣4 ⎦−2 1 −3⎡ ⎤ ⎡ ⎤ ⎡ ⎤−3 3 −2⎣ 30 ⎦ = 3⎣2⎦+6⎣4 ⎦ ♣−15 1 −3Matrix Form <strong>of</strong> a <strong>Linear</strong> SystemMatrix multiplication has an important application to system <strong>of</strong> linear equations Considerany system <strong>of</strong> m linear equations in n unknowns.a 11 x 1 +a 12 x 2 +···+a 1n x n = b 1a 21 x 1 +a 22 x 2 +···+a 2n x n = b 2 (1.11)....a m1 x 1 +a m2 x 2 +···+a mn x n = b mSince two matrices are equal if <strong>and</strong> only if their corresponding entries are equal, wecan replace the m equations in this system by the single matrix equation⎡⎤ ⎡ ⎤a 11 x 1 +a 12 x 2 +···+a 1n x n b 1a 21 x 1 +a 22 x 2 +···+a 2n x n⎢⎥⎣ . . . ⎦ = b 2⎢ ⎥⎣ . ⎦a m1 x 1 +a m2 x 2 +···+a mn x n b mThe m×1 matrix on the left side <strong>of</strong> this equation can be written as a product to give⎡ ⎤⎡⎤ ⎡ ⎤a 11 a 12 ··· a 1n x 1 b 1a 21 a 22 ··· a 2nx 2⎢ ⎥⎢⎥⎣ . . . ⎦⎣. ⎦ = b 2⎢ ⎥⎣ . ⎦ .a m1 a m2 ··· a mn x m b mIf we designate these matrices A, x, <strong>and</strong> b, respectively, then the original system <strong>of</strong> mequations in n unknowns has been replaced by the single matrix equationAx = b


Prepared by Dr.Archara Pacheenburawana (MA131 Sec 750001) 23The matrix A in this equation is called the coefficient matrix <strong>of</strong> the system. Theaugmented matrix for the system is obtained by adjoining b to A as the last column;thus the augmented matrix is⎡⎤a 11 a 12 ··· a 1n b 1[ ] a 21 a 22 ··· a 2n b 2A b = ⎢⎥⎣ . . . . ⎦ .a m1 a m2 ··· a mn b mTranspose <strong>of</strong> a MatrixDefinition 1.10 Transpose <strong>of</strong> a matrix. The transpose <strong>of</strong> the m×n matrix A, writtenA T , is the n×m matrix B whose jth column is the jth row <strong>of</strong> A. Equivalently, B = A Tif <strong>and</strong> only if b ij = a ji for each i <strong>and</strong> j.Example 1.16 Find the transpose <strong>of</strong>(a) A =[ ]2 7−8 0(b) B =[ ]1 2 −74 5 6(c) C = [ −1 3 4 ]Solution .........Definition 1.11 If A is a square matrix, then trace <strong>of</strong> A, denoted by tr(A), is definedto be the sum <strong>of</strong> the entries on the main diagonal <strong>of</strong> A. The trace <strong>of</strong> A is undefined if Ais not a square matrix.Example 1.17 Find the trace <strong>of</strong> matrices A <strong>and</strong> B.⎡ ⎤⎡ ⎤ −1 2 7 0a 11 a 12 a 13A = ⎣a 21 a 22 a 23⎦, B = ⎢ 3 5 −8 4⎥⎣ 1 2 7 −3⎦a 31 a 32 a 334 −2 1 0Solution .........Exercise 1.31. Suppose that A,B,C,D, <strong>and</strong> E are matrices with the following dimensions:A B C D E(4×5) (4×5) (5×2) (4×2) (5×4)Determine which <strong>of</strong> the following matrix expression are defined. For those that aredefined, give the dimension <strong>of</strong> the resulting matrix.(a) BA (b) AC +D (c) AE +B (d) AB +B(e) E(A+B) (f) E(AC) (g) E T A (h) (A T +E)D2. If a matrix A is 3×5 <strong>and</strong> the product AB is 3×7, what is the size <strong>of</strong> B?


Prepared by Dr.Archara Pacheenburawana (MA131 Sec 750001) 243. Solve the following matrix equation for a,b,c, <strong>and</strong> d[ ] [ ]a−b b+c 8 1=3d+c 2a−4d 7 64. Consider the matrices⎡ ⎤3 0A = ⎣−1 2⎦, B =1 1Compute the following (where possible).[ ] 4 −1, C =0 2⎡ ⎤ ⎡ ⎤1 5 2 6 1 3D = ⎣−1 0 1⎦, E = ⎣−1 1 2⎦3 2 4 4 1 3[ ] 1 4 2,3 1 5(a) 2A T +C (b) D T −E T (c) (D −E) T (d) B T +5C T(e) 1 2 CT − 1 4 A (f) B −BT (g) 2E T −3D T (h) (2E T −3D T ) T5. Using the matrices in Exercise 4, compute the following (where possible).(a) AB (b) BA (c) (3E)D (d) (AB)C(e) A(BC) (f) CC T (g) (DA) T (h) (C T B)A T(i) tr(DD T ) (j) tr(4E T −D) (k) tr(C T A T +2E T )[ ] [ ]1 −2 −1 2 −16. Let A = <strong>and</strong> AB = . Find the first <strong>and</strong> second columns−2 5 6 −9 3<strong>of</strong> B.7. LetUse the method <strong>of</strong> Example 1.13 to find⎡ ⎤ ⎡ ⎤1 2 1 1 0 2A = ⎣3 3 5⎦ <strong>and</strong> B = ⎣2 1 1⎦2 4 1 5 4 1(a) the first row <strong>of</strong> AB(c) the second column <strong>of</strong> AB(e) the third row <strong>of</strong> AA(b) the third row <strong>of</strong> AB(d) the first column <strong>of</strong> BA(f) the third column <strong>of</strong> AA8. Let A <strong>and</strong> B be the matrices Exercise7. Use the method <strong>of</strong> Example 1.15 to(a) express each column matrix <strong>of</strong> AB as a linear combination <strong>of</strong> the columnmatrices <strong>of</strong> A(b) express each column matrix <strong>of</strong> BA as a linear combination <strong>of</strong> the columnmatrices <strong>of</strong> B


Prepared by Dr.Archara Pacheenburawana (MA131 Sec 750001) 259. Ineachpart,expressthegivensystem<strong>of</strong>linearequationsasasinglematrixequationAx = b.(a) 3x 1 +2x 2 = 12x 1 −3x 2 = 5(b) x 1 + x 2 = 52x 1 + x 2 − x 3 = 63x 1 −2x 2 +2x 3 = 7(c) 2x 1 + x 2 + x 3 = 4x 1 − x 2 +2x 3 = 23x 1 −2x 2 − x 3 = 0(d) 4x 1 + −3x 3 + x 4 = 15x 1 + x 2 −8x 4 = 32x 1 −5x 2 +9x 3 − x 4 = 03x 2 − x 3 +7x 4 = 210. In each part, express the matrix equation as a system <strong>of</strong> linear equations.⎡ ⎤⎡⎤ ⎡ ⎤⎡ ⎤⎡⎤ ⎡ ⎤ 3 −2 0 1 w 03 −1 2 x 1 2(a) ⎣ 4 3 7⎦⎣x 2⎦ = ⎣−1⎦(b) ⎢ 5 0 2 −2⎥⎢x⎥⎣ 3 1 4 7 ⎦⎣y⎦ = ⎢0⎥⎣0⎦−2 1 5 x 3 4−2 5 1 6 z 011. In each part, find a 6×6 matrix [a ij ] that satisfies the state condition. Make youranswers as general as possible by using letters rather than specific numbers for thenonzero entries.(a) a ij = 0 if i ≠ j(b) a ij = 0 if i > j(c) a ij = 0 if i < j (d) a ij = 0 if |i−j| > 112. Find the matrix A such that⎡ ⎤ ⎡ ⎤x x+yA⎣y⎦ = ⎣x−y⎦z 0for all choices <strong>of</strong> x, y, <strong>and</strong> z?13. Indicate whether the statement is always true or sometimes false. Justify youranswer with a counterexample.(a) The expressions tr(AA T ) <strong>and</strong> tr(A T A) are always defined, regardless <strong>of</strong> thesize <strong>of</strong> A.(b) tr(AA T ) = tr(A T A) for every matrix A.(c) If the first column <strong>of</strong> A has all zeros, then so does the first column <strong>of</strong> everyproduct AB.(d) If the first row <strong>of</strong> A has all zeros, then so does the first row <strong>of</strong> every productAB.(e) If A is a square matrix with two identical rows, then AA has two identicalrows.(f) If A is a square matrix <strong>and</strong> AA has a column <strong>of</strong> zeros, then A must have acolumn <strong>of</strong> zeros.


Prepared by Dr.Archara Pacheenburawana (MA131 Sec 750001) 26(g) If B is an n×n matrix whose entries are positive even integers, <strong>and</strong> if A is ann×n matrix whose entries are positive integers, then the entries <strong>of</strong> AB <strong>and</strong>BA are positive even integers.(h) If the matrix sum AB +BA is defined, then A <strong>and</strong> B must be square.Answer to Exercise 1.31. (a) Undefined (b) 4×2 (c) Undefined (d) Undefined (e) 5×5 (f) 5×2(g) Undefined (h) 5×22. 5×7 3. a = 5,b = −3,c = 4,d = 1⎡ ⎤ ⎡ ⎤[ ] −5 0 −1 −5 0 −17 2 44. (a) (b) ⎣ 4 −1 1 ⎦ (c) ⎣ 4 −1 1 ⎦ (d) Undefined3 5 7−1 −1 1 −1 −1 1⎡ ⎤ ⎡ ⎤ ⎡ ⎤− 1 3 [ ] 9 1 −1 9 −13 04 2(e) ⎣ 90⎦ 0 −14(f) (g) ⎣−13 2 −4⎦ (h) ⎣ 1 2 1 ⎦3 9 1 00 1 −6 −1 −4 −64 4⎡ ⎤ ⎡ ⎤ ⎡ ⎤12 −342 108 75 3 45 95. (a) ⎣−4 5 ⎦ (b) Undefined (c) ⎣12 −3 21⎦ (d) ⎣11 −11 17⎦4 136 78 63 7 17 13⎡ ⎤⎡ ⎤3 45 9 [ ] [ ] 12 6 9(e) ⎣11 −11 17⎦ 21 17 0 −2 11(f) (g) (h) ⎣48 −20 14⎦17 35 12 1 87 17 1324 8 16[ [ ]7 −8(i) 61 (j) 35 (k) 28 6. b 1 = , b4]2 =−5⎡ ⎤ ⎡ ⎤⎡ ⎤7. (a) [ 10 6 5 ] (b) [ 15 8 9 ] 6 5(c) ⎣23⎦ (d) ⎣7⎦ (e) [ 16 20 23 ] 12(f) ⎣23⎦8 1923⎡ ⎤ ⎡ ⎤ ⎡ ⎤ ⎡ ⎤10 1 2 18. (a) ⎣34⎦ = 1⎣3⎦+2⎣3⎦+5⎣5⎦15 2 4 1⎡ ⎤ ⎡ ⎤ ⎡ ⎤ ⎡ ⎤6 1 2 1⎣23⎦ = 0⎣3⎦+1⎣3⎦+4⎣5⎦8 2 4 1⎡ ⎤ ⎡ ⎤ ⎡ ⎤ ⎡ ⎤5 1 2 1⎣14⎦ = 2⎣3⎦+1⎣3⎦+1⎣5⎦9 2 4 1⎡ ⎤ ⎡ ⎤ ⎡ ⎤ ⎡ ⎤5 1 0 2(b) ⎣7⎦ = 1⎣2⎦+3⎣1⎦+2⎣1⎦19 5 4 1


Prepared by Dr.Archara Pacheenburawana (MA131 Sec 750001) 279. (a)⎡ ⎤ ⎡ ⎤ ⎡ ⎤ ⎡ ⎤10 1 0 2⎣11⎦ = 2⎣2⎦+3⎣1⎦+4⎣1⎦26 5 4 1⎡ ⎤ ⎡ ⎤ ⎡ ⎤ ⎡ ⎤3 1 0 2⎣8⎦ = 1⎣2⎦+5⎣1⎦+1⎣1⎦26 5 4 1[ ][ ] [ 3 2 x1 1=2 −3 x 2 5]⎡2 1 1(c) ⎣1 −1 23 −2 −110. (a) 3x 1 − x 2 +2x 3 = 24x 1 +3x 2 +7x 3 = −1−2x 1 + x 2 +5x 3 = 4⎡ ⎤⎡⎤ ⎡ ⎤1 1 0 x 1 5(b) ⎣2 1 −1⎦⎣x 2⎦ = ⎣6⎦3 −2 2 x 3 7⎡ ⎤⎡⎤ ⎡ ⎤⎤⎡⎤ ⎡ ⎤ 4 0 −3 1 xx 1 41 1⎦⎣x 2⎦ = ⎣2⎦ (d) ⎢5 1 0 −8⎥⎢x 2⎥⎣2 −5 9 −1⎦⎣x x 3 03⎦ = ⎢3⎥⎣0⎦0 3 −1 7 x 4 2(b) 3w−2x + z = 05w +2y −2z = 03w+ x+4y +7z = 0−2w +5x+ y +6z = 0⎡⎤ ⎡⎤a 11 0 0 0 0 0 a 11 a 12 a 13 a 14 a 15 a 160 a 22 0 0 0 00 a 22 a 23 a 24 a 25 a 2611. (a)0 0 a 33 0 0 0⎢ 0 0 0 a 44 0 0(b)0 0 a 33 a 34 a 35 a 36⎥ ⎢ 0 0 0 a 44 a 45 a 46⎥⎣ 0 0 0 0 a 55 0 ⎦ ⎣ 0 0 0 0 a 55 a 56⎦0 0 0 0 0 a 66 0 0 0 0 0 a 66⎡⎤ ⎡⎤a 11 0 0 0 0 0 a 11 a 12 0 0 0 0a 21 a 22 0 0 0 0a 21 a 22 a 23 0 0 0(c)a 31 a 32 a 33 0 0 0⎢a 41 a 42 a 43 a 44 0 0(d)0 a 32 a 33 a 34 0 0⎥ ⎢ 0 0 a 43 a 44 a 45 0⎥⎣a 51 a 52 a 53 a 54 a 55 0 ⎦ ⎣ 0 0 0 a 54 a 55 a 56⎦a 61 a 62 a 63 a 64 a 65 a 66 0 0 0 0 a 65 a 66⎡ ⎤1 1 012. ⎣1 −1 0⎦0 0 0[ ] [ ] 0 2 1 213. (a) True (b) True (c) False; for example, A = <strong>and</strong> B =0 4 3 4[ ]1 −1(d) True (e) True (f) False; for example, A = (g) True (h) True1 −1


Prepared by Dr.Archara Pacheenburawana (MA131 Sec 750001) 281.4 Inverses; Rules <strong>of</strong> Matrix ArithmeticIn this section we shall discuss some properties <strong>of</strong> the arithmetic operations on matrices.We shall see that many <strong>of</strong> the basic rules <strong>of</strong> arithmetic for real numbers also hold formatrices, but a few do not.Properties <strong>of</strong> Matrix OperationsFor real numbers a <strong>and</strong> b, we always have ab = ba, which is called the commutative lawfor multiplication. For matrices, however, AB <strong>and</strong> BA need not be equal.Example 1.18 Consider the matrices[ ]1 0A = , B =2 3[ ]1 23 4Multiplying givesThus, AB ≠ BA ♣AB =[ ] [ ]1 2 5 6, BA =11 16 11 16Theorem 1.2 (Properties <strong>of</strong> <strong>Matrices</strong> Arithmetic) Let A, B, <strong>and</strong> C be matrices<strong>and</strong> a <strong>and</strong> b be scalars. Assume that the sizes <strong>of</strong> the matrices are such that each operationis defined. Then(a) A+B = B +A(Commutative law for addition)(b) A+(B +C) = (A+B)+C (Associative law for addition)(c) A(BC) = (AB)C(d) A(B +C) = AB +AC(e) (B +C)A = BA+CA(f) A(B −C) = AB −AC(h) a(B +C) = aB +aC(j) (a+b)C = aC +bC(l) (ab)C = a(bC)(Associative law for multiplication)(Left distributive laws)(Right distributive laws)(g) (B −C)A = BA−CA(i) a(B −C) = aB −aC(k) (a−b)C = aC −bC(m) a(BC) = (aB)C = B(aC)Example 1.19 Let⎡ ⎤1 0 −1⎡ ⎤−1 2 3⎡ ⎤−2 1 1A = ⎣2 1 −1⎦, B = ⎣ 0 1 0⎦, C = ⎣ 1 1 0 ⎦3 −2 0 −2 −1 0 3 0 −3Find A(BC), (AB)C, A(B +C), <strong>and</strong> AB +AC.Solution .........


Prepared by Dr.Archara Pacheenburawana (MA131 Sec 750001) 29Zero <strong>Matrices</strong>Definition 1.12 The matrix analog <strong>of</strong> the number 0 is called a zero matrix. A zeromatrix, denoted by 0, is a matrix <strong>of</strong> any sizes consisting <strong>of</strong> only zero entries.For example, the matrices⎡ ⎤0 0 0⎣0 0 0⎦,0 0 0[ ] 0 0 0,0 0 0⎡ ⎤0⎣0⎦, <strong>and</strong>0[ 0 0 0 0]are all zero matrices.Since we already know that some <strong>of</strong> the rules <strong>of</strong> arithmetic for real number do notcarryover to matrix arithmetic. Forexample, consider thefollowing two st<strong>and</strong>ardresultsin the arithmetic <strong>of</strong> real numbers.• If ab = ac <strong>and</strong> a ≠ 0, then b = c. (This is called the cancellation law.)• If ad = 0, then at least one <strong>of</strong> the factors on the left is 0As the next example shows, the corresponding results are not generally true in matrixarithmetic.Example 1.20 Consider the matrices[ ] [ ]0 1 1 1A = , B = , C =0 2 3 4ThenAB = AC =[ ]3 46 8[ ]2 5, D =3 4AD =[ ]0 00 0[ ]3 70 0Thus, although A ≠ 0, it is incorrect to cancel the A from both sides <strong>of</strong> the equationAB = AC <strong>and</strong> write B = C. Also, AD = 0, yet A ≠ 0 <strong>and</strong> D ≠ 0. Thus, thecancellation law is not valid for matrix multiplication, <strong>and</strong> it is possible for a product <strong>of</strong>matrices to be zero without either factor being zero. ♣Theorem 1.3 (Properties <strong>of</strong> Zero <strong>Matrices</strong>) Assuming that the sizes <strong>of</strong> the matricesare such that the indicated operations can be performed, the following rules <strong>of</strong> matrixarithmetic are valid.(a) A+0 = 0+A = A(b) A−A = 0(c) 0−A = −A(d) A0 = 0; 0A = 0


Prepared by Dr.Archara Pacheenburawana (MA131 Sec 750001) 30Identity <strong>Matrices</strong>Of special interest are square matrices with 1’s on the main diagonal <strong>and</strong> 0’s <strong>of</strong>f the maindiagonal, such as⎡ ⎤⎡ ⎤[ ]1 0 0 01 0 0 1 0, ⎣0 1 0⎦,⎢0 1 0 0⎥0 1 ⎣0 0 1 0⎦ .0 0 10 0 0 1Amatrix<strong>of</strong>thisformiscalledanidentity matrix <strong>and</strong>isdenotedbyI. Ifitisimportantto emphasize the size, we shall write I n for the n×n identity matrix.The identity matrix is so named because it has the property that AI n = A <strong>and</strong>I m A = A for any m × n matrix A. Thus it is the matrix analog <strong>of</strong> the number 1, themultiplicative identity for numbers.Example 1.21 Consider the matrices⎡ ⎤ ⎡ ⎤3 4 1 b 11 b 12A = ⎣2 6 3⎦ B = ⎣b 21 b 22⎦0 1 8 b 31 b 32Then<strong>and</strong>⎡ ⎤⎡⎤ ⎡ ⎤1 0 0 3 4 1 3 4 1I 3 A = ⎣0 1 0⎦⎣2 6 3⎦ = ⎣2 6 3⎦ = A0 0 1 0 1 8 0 1 8⎡ ⎤ ⎡ ⎤b 11 b 12[ ] bBI 2 = ⎣b 21 b 22⎦ 1 0 11 b 12= ⎣b 0 1 21 b 22⎦ = B ♣b 31 b 32 b 31 b 32Theorem 1.4 If R is the reduced row-echelon form <strong>of</strong> an n×n matrix A, then eitherR has a row <strong>of</strong> zeros or R is the identity matrix I n .Definition 1.13 Let A be a square matrix. If there exist a matrix B such that AB =BA = I, we say that A is invertible (or nonsingular) <strong>and</strong> that B is the inverse <strong>of</strong>A. If A has no inverse, it is said to be noninvertible (or singular).Example 1.22 Show that the matrix B =Solution .........Properties <strong>of</strong> Inverses[3 51 2]is an inverse <strong>of</strong> A =Theorem 1.5 If B <strong>and</strong> C are both inverses <strong>of</strong> the matrix A, then B = C.[ ]2 −5.−1 3Asaconsequence <strong>of</strong>Theorem 1.5, wecansaynowspeak<strong>of</strong> the inverse <strong>of</strong>aninvertiblematrix. If A is invertible, then its inverse will be denoted by the symbol A −1 . Thus,AA −1 = I <strong>and</strong> A −1 A = I


Prepared by Dr.Archara Pacheenburawana (MA131 Sec 750001) 31Theorem 1.6 The matrixA =[ ]a bc dis invertible if ad−bc ≠ 0, in which case the inverse is given by the formulaA −1 =1ad−bc[ ] d −b=−c a⎡⎢⎣dad−bc− cad−bc− b ⎤ad−bc⎥a ⎦.ad−bcTheorem 1.7 If A <strong>and</strong> B are invertible matrices <strong>of</strong> the same dimension, then AB isinvertible <strong>and</strong>(AB) −1 = B −1 A −1Theorem 1.8 If A 1 ,A 2 ,...,A n are invertible matrices <strong>of</strong> the same dimension, thenA 1 A 2···A n is invertible <strong>and</strong>Example 1.23 Let A =Solution .........Powers <strong>of</strong> a Matrix(A 1 A 2···A n ) −1 = A −1n A−1 n−1···A−1 2 A−1 1][ ] 7 −3<strong>and</strong> B =5 −2[ 4 13 1. Show that (AB) −1 = B −1 A −1 .Definition 1.14 If A is a square matrix, then we define the nonnegative integer powers<strong>of</strong> A to beA 0 = I A n = AA···A } {{ } (n > 0)nfactorsMoreover, if A is invertible, then we define the negative integer powers to beA −n = (A −1 ) n = A −1 −1} A−1···A {{ }nfactorsNote In general, if A is a square matrix, <strong>and</strong> r <strong>and</strong> s are positive integers, we haveA r A s = A r+s . It is also true that under the same condition, (A r ) s = A rs .[ ] [ ]3 −2 1 2Example 1.24 Let A = <strong>and</strong> A−1 1−1 = . Find the matrices A1 33 <strong>and</strong> A −3 .Solution .........Theorem 1.9 (Laws <strong>of</strong> Exponents) If A is invertible matrix, then:(a) A −1 is invertible <strong>and</strong> (A −1 ) −1 = A .(b) A n is invertible <strong>and</strong> (A n ) −1 = (A −1 ) n for n = 0,1,2,....


Prepared by Dr.Archara Pacheenburawana (MA131 Sec 750001) 32(c) For any nonzero scalar k, the matrix kA is invertible <strong>and</strong> (kA) −1 = 1 k A−1 .Example 1.25 If( 13 A ) −1=Solution .........Properties <strong>of</strong> the Transpose[ ] 2 −1, then find A.−5 3Theorem 1.10 If the sizes <strong>of</strong> the matrices are such that the state operations can beperformed, then(a) (A T ) T = A(b) (A+B) T = A T +B T <strong>and</strong> (A−B) T = A T −B T(c) (kA) T = kA T , where k is any scalar(d) (AB) T = B T A TInvertibility <strong>of</strong> a TransposeTheorem 1.11 If A is an invertible matrix, then A T is also invertible <strong>and</strong>(A T ) −1 = (A −1 ) T .[ ]−3 1Example 1.26 Let A = . Show that (A5 −2T ) −1 = (A −1 ) T .Solution .........1. LetShow that(a) A+(B +C) = (A+B)+C(c) (a+b)C = aC +bC(e) a(BC) = (aB)C = B(aC)(g) (B +C)A = BA+CA(i) (A T ) T = A(k) (aC) T = aC TExercise 1.4⎡ ⎤ ⎡ ⎤1 −2 0 6 0 2A = ⎣3 5 −1⎦, B = ⎣ 4 1 −1⎦2 3 4 −3 8 5⎡ ⎤4 −5 3C = ⎣ 5 7 −2⎦, a = 3, b = −5−3 2 −1(b) (AB)C = A(BC)(d) a(B −C) = aB −aC(f) A(B −C) = AB −AC(h) a(bC) = (ab)C(j) (A+B) T = A T +B T(l) (AB) T = B T A T


Prepared by Dr.Archara Pacheenburawana (MA131 Sec 750001) 332. Let 0 denote a 2×2 matrix, each <strong>of</strong> whose entries is zero. Find(a) 2×2 matrix A such that A ≠ 0 <strong>and</strong> AA = 0 <strong>and</strong>(b) 2×2 matrix A such that A ≠ 0 <strong>and</strong> AA = A.3. Use Theorem 1.6 to compute the inverses <strong>of</strong> the following matrices.[ ] 1 3(a) A =2 −4[ ]−4 −5(c) C =5 6[ ] 2 3(b) B =1 1[ ]3 −7(d) D =−6 134. Use the matrices A, B, <strong>and</strong> C in Exercise3 to verify that(a) (A −1 ) −1 = A(b) (B T ) −1 = (B −1 ) T(c) (AB) −1 = B −1 A −1 (d) (ABC) −1 = C −1 B −1 A −1[ ] 1 −15. Let A = . Find the power A−1 22 <strong>and</strong> A 3 .⎡ ⎤1 0 06. Let A = ⎣2 1 0⎦. Find the power A 2 <strong>and</strong> A 3 .3 2 1[ ]1 17. Let A = . Find a general formula for A0 1n .8. In each part, use the given information to find A.[ ][ ]2 −1−3 7(a) A −1 =(b) (7A)3 5−1 =1 −2[ ][ ]−3 −1−1 2(c) (5A T ) −1 =(d) (I +2A)5 2−1 =4 5[ ] cosθ sinθ9. Find the inverse <strong>of</strong> .−sinθ cosθ[ 1]10. Find the inverse <strong>of</strong>2 (ex +e −x 1)2 (ex −e −x )12 (ex −e −x 1)2 (ex +e −x .)11. Consider the matrix⎡ ⎤a 11 0 ··· 00 a 22 ··· 0A = ⎢ ⎥⎣ . . . ⎦0 0 ··· a nnwhere a 11 a 22···a nn ≠ 0. Show that A is invertible <strong>and</strong> find its inverse.


Prepared by Dr.Archara Pacheenburawana (MA131 Sec 750001) 3412. Let A, B, <strong>and</strong> 0 be 2×2 matrices. Assuming that A is invertible, find a matrix Csuch that [ A−10C A −1 ]is the inverse <strong>of</strong> the partitioned matrix[ A 0B A13. Use the result in Exercise 12 to find the inverses <strong>of</strong> the following matrices.]⎡ ⎤1 1 0 0(a) ⎢−1 1 0 0⎥⎣ 1 1 1 1⎦1 1 −1 1⎡ ⎤1 1 0 0(b) ⎢0 1 0 0⎥⎣0 0 1 1⎦0 0 0 114. (a) Find a nonzero 3×3 matrix A such that A is symmetric.(b) Find a nonzero 3×3 matrix A such that A is skew-symmetric.15. A square matrix A is called symmetric if A T = A <strong>and</strong> skew-symmetric ifA T = −A. Show that if B is a square matrix, then(a) BB T <strong>and</strong> B +B T are symmetric,(b) B −B T is skew-symmetric.16. Indicate whether the statement is always true or sometimes false. Justify youranswer with a counterexample.(a) (AB) 2 = A 2 B 2 , where A are B are n×n matrices.(b) (A−B) 2 = (B −A) 2 , where A are B are n×n matrices.(c) (A+B)(A−B) = A 2 −B 2 , where A are B are n×n matrices.(d) (AB −1 )(BA −1 ) = I n , where A are B are n×n matrices.(e) Let A <strong>and</strong> B be square matrices such that AB = 0. Show that if A isinvertible, then B = 0.(f) If a square matrix A satisfies A 2 −3A+I = 0, then A −1 = 3I −A.(g) If A <strong>and</strong> B are m×n matrices, then AB T <strong>and</strong> A T B are defined.(h) If AB = BA <strong>and</strong> if A is invertible, then A −1 B = BA −1 .(i) If A T is singular, then A is singular.(j) A matrix with a row <strong>of</strong> zeros cannot have an inverse.(k) A matrix with a column <strong>of</strong> zeros cannot have an inverse.(l) If A is invertible <strong>and</strong> AB = AC, then B = C.(m) Let A be an n×n matrix <strong>and</strong> let x <strong>and</strong> y be n×1 matrices. If Ax = Ay <strong>and</strong>x ≠ y, then A is singular.


Prepared by Dr.Archara Pacheenburawana (MA131 Sec 750001) 352. (a)(n) IfAisinvertible <strong>and</strong>k isanynonzero scalar, then(kA) n = k n A n forall integervalues <strong>of</strong> n.(o) Sum <strong>of</strong> two invertible matrices is invertible.(p) If AB is invertible, then B is invertible.[ ] [ ] a b x y(q) + is symmetric.b a y x[ ]0 10 03. (a) A −1 =(d) D −1 =[ ]1 0(b)0 0][2 35 101− 15 10[−133− 7 3−2 −1]Answer to Exercise 1.4(b) B −1 =[ ]−1 31 −2(c) C −1 =[ ] [ ]2 −3 5 −85. A 2 = , A−3 53 =−8 13⎡ ⎤ ⎡ ⎤1 0 0 1 0 06. A 2 = ⎣4 1 0⎦, A 3 = ⎣6 1 0⎦ 7. A n =10 4 1 21 6 1[ ] [ ] [ 5 1 2113 13 7 −28. (a) A = (b) A = (c) A =(d) A =− 313[−9213113 13213− 613]1737[ ]1 n0 1[ ] [ cosθ −sinθ 1]9. 102 (ex +e −x ) − 1 2 (ex −e −x )sinθ cosθ − 1 2 (ex −e −x 1)2 (ex +e −x )⎡ ⎤1a 110 ··· 010a11. A = ⎢22··· 0⎥⎣ . . . ⎦ 12. C = −A−1 BA −110 0 ···a nn⎡ ⎤1− 1 0 0 ⎡ ⎤2 2 1 11 −1 0 00 013. (a)2 2 ⎢ 1⎣ 0 0 − 1 ⎥2 2⎦ (b) ⎢0 1 0 0⎥⎣0 0 1 −1⎦1 1 0 0 0 1−1 02 2− 1 51535][ ]6 5−5 −4


Prepared by Dr.Archara Pacheenburawana (MA131 Sec 750001) 36⎡ ⎤ ⎡ ⎤1 2 3 0 −1 −114. (a) ⎣2 1 4⎦ (b) ⎣1 0 −1⎦3 4 5 1 1 0[ ] [ ] 0 1 1 216. (a) False; for example, A = <strong>and</strong> B =0 2 3 4[ ] [ ]0 1 1 2(c) False; for example, A = <strong>and</strong> B =0 2 3 4(b) True(d) True (e) True(f) True (g) True (h) True (i) True (j) True (k) True (l) True (m) True[ ] [ ]1 0 −1 0(n) True (o) False; for example, A = <strong>and</strong> B = (p) True0 1 0 −1(q) True1.5 Elementary <strong>Matrices</strong> <strong>and</strong> a Method for FindingA −1In this section we shall develop an algorithm for finding the inverse <strong>of</strong> an invertiblematrix. We shall also discuss some <strong>of</strong> the basic properties <strong>of</strong> invertible matrices.Definition 1.15 An n×n matrix is called an elementary matrix if it can be obtainedfrom the n×n identity matrix I n by performing a single elementary row operation.List below are three elementary matrices <strong>and</strong> the operations that produce them.[ ] 1 0Multiply the second row <strong>of</strong> I0 −52 by −5.⎡ ⎤1 0 0 0⎢0 0 0 1⎥⎣0 0 1 0⎦ Interchange the second <strong>and</strong> fourth rows <strong>of</strong> I 4.0 1 0 0⎡ ⎤1 0 −2⎣0 1 0 ⎦ Add −2 times the third row <strong>of</strong> I 3 to the first row.0 0 1Theorem 1.12 If the elementary matrix E results from performing a certain row operationon I m <strong>and</strong> A is an m×n matrix, then the product EA is the matrix that resultswhen this same row operation is performed on A.Example 1.27 Let⎡ ⎤ ⎡ ⎤ ⎡ ⎤1 0 0 0 1 0 1 0 0E 1 = ⎣ 0 1 0⎦, E 2 = ⎣1 0 0⎦, E 3 = ⎣0 1 0⎦−4 0 1 0 0 1 0 0 5


Prepared by Dr.Archara Pacheenburawana (MA131 Sec 750001) 37<strong>and</strong>Find the product E 1 A, E 2 A, <strong>and</strong> E 3 A.Solution .........⎡ ⎤a b cA = ⎣d e f⎦g h iIf an elementary row operation is applied to an identity matrix I to produce anelementary matrix E, then there is a second row operation that, when applied to E,produces I back again. For example, if E is obtained by multiplying the ith row <strong>of</strong> Iby a nonzero constant k, then I can be recovered if the ith row <strong>of</strong> E is multiplied by1. The various possibilities are listed in the following table. The operations on the rightkside <strong>of</strong> this table are called the inverse operations <strong>of</strong> the corresponding operations onthe left.Row Operation on IThat Produces EMultiply row i by k ≠ 0Row Operation on EThat Reproduces IMultiply row i by 1 kInterchange rows i <strong>and</strong> j Interchange rows i <strong>and</strong> jAdd k times row i to row j Add −k times row i to row jExample 1.28 In each <strong>of</strong> the following, an elementary row operation is applied to the2×2 identity matrix to obtain an elementary matrix E, then E is restored to the identitymatrix by applying the inverse row operation.[ 1 00 1[1 00 1]−→↑Multiply thesecond row by 3]−→↑Interchange the first<strong>and</strong> second rows[ 1 00 1]−→↑Add −7 times thesecond row tothe first[ 1 00 3[0 11 0]−→[ ] 1 00 1↑Multiply thesecond row by 1 3][ 1 −70 1−→[1 00 1]↑Interchange the first<strong>and</strong> second rows]−→[ 1 00 1↑Add −7 times thesecond row tothe firstThe next theorem gives an important property <strong>of</strong> elementary matrices.Theorem 1.13 Every elementary matrix is invertible, <strong>and</strong> the inverse is also an elementarymatrix.]


Prepared by Dr.Archara Pacheenburawana (MA131 Sec 750001) 38Theorem 1.14 If A is an n×n matrix, then the following statements are equivalent,that is, all true or all false.(a) A is invertible.(b) Ax = 0 has only the trivial solution.(c) The reduced row-echelon form <strong>of</strong> A is I n .(d) A is expressible as a product <strong>of</strong> elementary matrices.By Theorem 1.14(c), we can find elementary matrices E 1 , E 2 , ..., E k−1 , E k such thatE k E k−1···E 2 E 1 A = I n (1.12)By Theorem 1.13, E 1 , E 2 , ..., E k−1 , E k are invertible. Multiplying both sides <strong>of</strong>Equation (1.12) on the left successively by E −1k , E−1 k−1, ..., E−1 2 , E1 −1 we obtainA = E1 −1 E2 −1 ···E −1k−1 E−1 k I n = E1 −1 E2 −1 ···E −1k−1 E−1 kBy Theorem 1.13, this equation expresses A as a product <strong>of</strong> elementary matrices.Row EquivalenceIf a matrix B can be obtained from a matrix A by performing a finite sequence <strong>of</strong>elementary row operations, then obviously we can get from B back to A by performingthe inverses <strong>of</strong> these elementary row operations in reverse order. <strong>Matrices</strong> that can beobtained from one another by a finite sequence <strong>of</strong> elementary row operations are said tobe row equivalent.In other words, a matrix B is row equivalent to A if there exists a finite sequence E 1 ,E 2 , ..., E k−1 , E k <strong>of</strong> elementary matrices such thatExample 1.29 LetB = E k E k−1···E 2 E 1 A.⎡ ⎤1 2 −3A = ⎣1 −2 1 ⎦.5 −2 −3Find the elementary matrices E 1 , E 2 , ..., E k−1 , E k such thatE k E k−1···E 2 E 1 A = A rwhere A r is the reduced row-echelon form <strong>of</strong> A.Solution .........


Prepared by Dr.Archara Pacheenburawana (MA131 Sec 750001) 39A Method for Inverting <strong>Matrices</strong>As our first application <strong>of</strong> Theorem 1.14, we shall establish a method for determiningthe inverse <strong>of</strong> an invertible matrix. Multiplying (1.12) on the right by A −1 yields(E k E k−1···E 2 E 1 A)A −1 = I n A −1E k E k−1···E 2 E 1 I n = A −1which tells us that A −1 can be obtained by multiplying I n successively on the left by theelementary matrices E 1 , E 2 , ..., E k−1 , E k . Since each multiplication on the left by one<strong>of</strong> these elementary matrices performs a row operation, it follow that the sequence <strong>of</strong> rowoperations that reduces A to I n will reduce I n to A −1 . Thus we have the following result:To find the inverse <strong>of</strong> an invertible matrix A we must find a sequence <strong>of</strong>elementary row operations that reduces A to identity <strong>and</strong> then performthis sequence <strong>of</strong> operation on I n to obtain A −1To accomplish this we shall adjoint the identity matrix to the right side <strong>of</strong> A, therebyproducing a matrix <strong>of</strong> the form[A|I].Then we shall apply row operations to this matrix until the left side is reduced to I;these operations will convert the right side to A −1 , so the final matrix will have the form[I|A −1 ]Example 1.30 Find the inverse <strong>of</strong>⎡ ⎤1 −2 1A = ⎣2 −5 2 ⎦3 2 −1Solution .........Often it will not be known in advance whether a given matrix is invertible. If an n×n(matrix A is not invertible,)then it cannot be reduced to I n by elementary row operationpart (c) <strong>of</strong> Theorem 1.14 . Stated another way, the reduced row-echelon form <strong>of</strong> A hasat least one row <strong>of</strong> zeros. Thus, if the procedure in the last example is attempted on amatrix that is not invertible, then at some point in the computations a row <strong>of</strong> zeros willoccur on the left side. It can then be concluded that the given matrix is not invertible,<strong>and</strong> the computations can be stopped.⎡ ⎤1 3 4Example 1.31 Let A be the matrix ⎣−2 −5 −3⎦. Determine whether A is invertible.1 4 9Solution .........


Prepared by Dr.Archara Pacheenburawana (MA131 Sec 750001) 40Example 1.32 In Example 1.30 we show that⎡ ⎤1 −2 1A = ⎣2 −5 2 ⎦3 2 −1is an invertible matrix. From Theorem 1.14, it follows that the homogeneous systemhas only the trivial solution. ♣x 1 −2x 2 + x 3 = 02x 1 −5x 2 +2x 3 = 03x 1 +2x 2 − x 3 = 0Exercise 1.51. Which <strong>of</strong> the following are elementary matrices?[ ] [ ] [ ]0 1 −3 1 1 0(a) (b) (c)1 0 1 0 0 √ 2⎡ ⎤⎡ ⎤ ⎡ ⎤ 2 0 0 21 1 0 0 1 0(e) ⎣0 0 1⎦ (f) ⎣1 0 0⎦ (g) ⎢0 1 0 0⎥⎣0 0 1 0⎦0 0 0 0 0 10 0 0 1⎡ ⎤1 0 0(d) ⎣0 1 0⎦5 0 12. Find a row operation that will restore the given elementary matrix to an identitymatrix.⎡ ⎤[ ]1 0 01 0(a)(b) ⎣0 1 0⎦−3 10 0 5⎡ ⎤1 0 0 0(c) ⎢0 0 0 1⎥⎣0 0 1 0⎦0 1 0 0⎡ ⎤1 0 − 1 04 (d) ⎢0 1 0 0⎥⎣0 0 1 0⎦0 0 0 13. For each <strong>of</strong> the following pairs <strong>of</strong> matrices, find an elementary matrix E such thatEA = B.


Prepared by Dr.Archara Pacheenburawana (MA131 Sec 750001) 41[ ] [ ]2 −1 −4 2(a) A =B =5 35 3⎡ ⎤2 1 3(b) A = ⎣−2 4 5⎦ ⎡ ⎤2 1 3B = ⎣ 3 1 4⎦3 1 4 −2 4 5⎡ ⎤4 −2 3⎡ ⎤4 −2 3(c) A = ⎣ 1 0 2⎦ B = ⎣1 0 2⎦−2 3 1 0 3 5⎡3 4⎤1⎡3 4⎤1(d) A = ⎣2 −7 −1⎦ B = ⎣2 −7 −1⎦2 −7 3 8 1 54. Given⎡ ⎤ ⎡ ⎤ ⎡ ⎤1 2 4 1 2 4 1 2 4A = ⎣2 1 3⎦, B = ⎣2 1 3⎦, C = ⎣0 −1 −3⎦1 0 2 2 2 6 2 2 6(a) Find an elementary matrix E such that EA = B.(b) Find an elementary matrixF such that FB = C.(c) Is C row equivalent to A? Explain.5. Given⎡ ⎤2 1 1A = ⎣6 4 5⎦4 1 3Find elementary matrices E 1 , E 2 , E 3 such thatwhere U is an upper triangular matrix.E 3 E 2 E 1 A = U6. Use the method shown in Example 1.30 <strong>and</strong> Example 1.31 to find the inverse <strong>of</strong>the given matrix if the matrix is invertible.[ ] [ ] [ ]1 4 −3 6 6 −4(a) (b)(c)2 7 4 5 −3 2⎡ ⎤1 0 1(d) ⎣3 3 4⎦⎡ ⎤1 1 1(e) ⎣0 1 1⎦⎡ ⎤2 0 5(f) ⎣0 3 0⎦2 2 3 0 0 1 1 0 3⎡ ⎤ ⎡ ⎤ ⎡ √ √ ⎤1 12 1 4− 2 5 5 5 2 3 2 0(g) ⎣3 2 5⎦(h) ⎣11 1 ⎦5 5 10(i) ⎣−4 √2 2 0 ⎦0 −1 10 0 115− 4 5110


Prepared by Dr.Archara Pacheenburawana (MA131 Sec 750001) 42⎡ ⎤1 0 0 0(j) ⎢1 3 0 0⎥⎣1 3 5 0⎦1 3 5 7⎡ ⎤1−8 17 232(k) ⎢ 4 0 −95 ⎥⎣ 0 0 0 0 ⎦−1 13 4 2⎡ ⎤0 0 2 0(l) ⎢1 0 0 1⎥⎣0 −1 3 0 ⎦2 1 5 −37. Find all values <strong>of</strong> a such that the following matrices are invertible:⎡ ⎤[ ]1 a 02 a(a) A =(b) A = ⎣−1 0 1⎦3 40 1 18. Givencompute A −1 <strong>and</strong> use it to:A =[ ] 3 15 2B =[ ] 1 23 4(a) Find a 2×2 matrix X such that AX = B.(b) Find a 2×2 matrix Y such that YA = B.9. Consider the matrixA =[ ] 1 0−5 2(a) Find elementary matrices E 1 <strong>and</strong> E 2 such that E 2 E 1 A = I.(b) Write A −1 as a product <strong>of</strong> two elementary matrices.(c) Write A as a product <strong>of</strong> two elementary matrices.10. Express the matrix⎡0⎤1 7 8A = ⎣ 1 3 3 8 ⎦−2 −5 1 −8in the form A = EFGR, where E,F, <strong>and</strong> G are elementary matrices <strong>and</strong> R is inrow-echelon form.11. Consider the following given matrix A. Findelementary matrices E 1 , E 2 , ..., E k−1 ,E k such that E k E k−1···E 2 E 1 A = A r where A r is the reduced row-echelon form <strong>of</strong>A.⎡ ⎤⎛ ⎞1 0 2(a) ⎣0 1 0⎦0 0 3⎛ ⎞0 0 1 1(c) ⎝1 2 0 0⎠3 0 0 12 0 2 4(b) ⎝1 1 2 −10 0 2 4⎠⎡ ⎤1 1 −1 −1(d) ⎢2 0 0 2⎥⎣0 0 0 3 ⎦2 2 −2 −212. Indicate whether the statement is always true or sometimes false.


Prepared by Dr.Archara Pacheenburawana (MA131 Sec 750001) 43(a) Every square matrix can be expressed as a product <strong>of</strong> elementary matrices.(b) The product <strong>of</strong> two elementary matrices is an elementary matrix.(c) If A is invertible <strong>and</strong> a multiple <strong>of</strong> the first row <strong>of</strong> A is added to the secondrow, then the resulting matrix is invertible.(d) If A is invertible <strong>and</strong> AB = 0, then it must be true that B = 0.(e) If A is a singular n×n matrix, then Ax = 0 has infinitely many solutions.(f) If A is a singular n×n matrix, then the reduced row-echelon from <strong>of</strong> A hasat least one row <strong>of</strong> zeros.(g) If A −1 is expressible as a product <strong>of</strong> elementary matrices, then the homogeneouslinear system Ax = 0 has only the trivial solution.Answer to Exercise 1.51. (a), (c), (d), (f) 2. (a) r 2 +3r 1 (b) 5r 3 (c) r 2 ↔ r 4 (d) r 1 + 1 r 4 3⎡ ⎤ ⎡ ⎤ ⎡ ⎤[ ] 1 0 0 1 0 0 1 0 0−2 03. (a) (b) ⎣0 0 1⎦ (c) ⎣0 1 0⎦ (d) ⎣0 1 0⎦0 10 1 0 0 2 1 2 0 1⎡ ⎤1 0 0⎡ ⎤1 0 04. (a) E = ⎣0 1 0⎦ (b) F = ⎣0 1 −1⎦1 0 1 0 0 1⎡ ⎤1 0 0⎡ ⎤1 0 0⎡ ⎤1 0 05. E 1 = ⎣−3 1 0⎦, E 2 = ⎣ 0 1 0⎦, E 3 = ⎣0 1 0⎦0 0 1 −2 0 1 0 1 1[ ] [ ]⎡ ⎤−7 4 −5 21 2 −339 136. (a) (b) (c) Not invertible (d) ⎣−1 1 −1⎦2 −1 4 139 130 −2 3⎡ ⎤ ⎡ ⎤ ⎡ ⎤1 −1 0 3 0 −5 −7 5 3(e) ⎣0 1 −1⎦ (f) ⎣ 10 0 ⎦3(g) ⎣ 3 −2 −2⎦0 0 1 −1 0 2 3 −2 −1⎡ ⎤ ⎡ √1 3 1 2 −3 √ ⎤ ⎡ ⎤2 1 0 0 0026 26(h) ⎣ 0 1 −1⎦ ⎢(i) ⎣4 √2 2⎥0⎦ (j) ⎢− 10 03 3 ⎥26 26 ⎣ 0 −−2 2 010⎦5 50 0 1 0 0 − 1 17 7⎡ ⎤(k) Not invertible (l)⎢⎣− 4 537. (a) a ≠ 8 3(b) a ≠ 1 8. (a)3515150 −1 0210 0 024 2− 1 − 1 5 5 5 5][−1 04 2⎥⎦(b)[ ]−8 5−14 9


Prepared by Dr.Archara Pacheenburawana (MA131 Sec 750001) 44[ ] [ ]1 0 1 09. (a) E 1 = , E5 1 2 =0 1 (b) A −1 = E 2 E 1 (c) A = E1 −1 E2−12⎡ ⎤⎡⎤⎡⎤⎡⎤0 1 0 1 0 0 1 0 0 1 3 3 810. ⎣1 0 0⎦⎣0 1 0⎦⎣0 1 0⎦⎣0 1 7 8⎦0 0 1 −2 0 1 0 1 1 0 0 0 0⎡ ⎤ ⎡ ⎤1 0 0 1 0 −211. (a) E 1 = ⎣0 1 0⎦, E 2 = ⎣0 1 0 ⎦0 0 1 0 0 13⎡ ⎤ ⎡ ⎤ ⎡ ⎤10 0 1 0 0 1 0 02(b) E 1 = ⎣0 1 0⎦, E 2 = ⎣−1 1 0⎦, E 3 = ⎣0 1 0⎦,0 1 0 0 0 1 0 0 1 2⎡ ⎤ ⎡ ⎤1 0 −1 1 0 0E 4 = ⎣0 1 0 ⎦, E 5 = ⎣0 1 −1⎦0 0 1 0 0 1⎡ ⎤ ⎡ ⎤ ⎡ ⎤0 1 0 1 0 0 1 0 0(c) E 1 = ⎣1 0 0⎦, E 2 = ⎣ 0 1 0⎦, E 3 = ⎣0 0 1⎦,0 0 1 −3 0 1 0 1 0⎡ ⎤ ⎡ ⎤1 0 0 1 −2 0E 4 = ⎣0 − 1 0⎦, E6 5 = ⎣0 1 0⎦0 0 1 0 0 1⎡ ⎤ ⎡ ⎤ ⎡ ⎤1 0 0 0 1 0 0 0 1 0 0 0(d) E 1 = ⎢−2 1 0 0⎥⎣ 0 0 1 0⎦ , E 2 = ⎢ 0 1 0 0⎥⎣ 0 0 1 0⎦ , E 3 = ⎢0 − 1 0 02 ⎥⎣0 0 1 0⎦0 0 0 1 −2 0 0 1 0 0 0 1⎡ ⎤ ⎡ ⎤ ⎡ ⎤1 −1 0 0 1 0 0 0 1 0 −1 0E 4 = ⎢0 1 0 0⎥⎣0 0 1 0⎦ , E 5 = ⎢0 1 0 0⎥⎣0 0 1 0⎦ , E 6 = ⎢0 1 0 0⎥⎣0 0 1 0⎦ ,30 0 0 1 0 0 0 1 0 0 0 1⎡ ⎤1 0 0 0E 7 = ⎢0 1 2 0⎥⎣0 0 1 0⎦0 0 0 112. (a) False (b) True (c) True (d) True (e) True (f) True (g) True1.6 Further Results on <strong>Systems</strong> <strong>of</strong> <strong>Equations</strong> <strong>and</strong>InvertibilityA Basic TheoremTheorem 1.15 Every system <strong>of</strong> linear equations has no solutions, or has exactly onesolutions, or has infinitely many solutions.


Prepared by Dr.Archara Pacheenburawana (MA131 Sec 750001) 45Solving <strong>Linear</strong> <strong>Systems</strong> by Matrix InversionSo far, we have studied two methods for solving linear system: Gaussian elimination <strong>and</strong>Gauss-Jordan elimination. The following theorem provides a new method for solvingcertain linear systems.Theorem 1.16 If A is an invertible n × n matrix, then for each n × 1 matrix b, thesystem <strong>of</strong> equations Ax = b has exactly one solution, namely, x = A −1 b.Example 1.33 Solve the systemSolution .........x 1 −2x 2 + x 3 = 72x 1 −5x 2 +2x 3 = 63x 1 +2x 2 − x 3 = 1Remark Note that the method <strong>of</strong> Example 1.33 applies only when the system has asmany equations as unknowns, <strong>and</strong>the coefficient matrixis invertible. This methodis lessefficient, computationally, than Gaussian elimination, but it is important in the analysis<strong>of</strong> equations involving matrices.<strong>Linear</strong> <strong>Systems</strong> with a Common Coefficient MatrixFrequently, one is concerned with solving a sequence <strong>of</strong> systemsAx = b 1 , Ax = b 2 , Ax = b 3 ,..., Ax = b keach <strong>of</strong> which has the same square coefficient matrix A. If A is invertible, then thesolutionsx 1 = A −1 b 1 , x 2 = A −1 b 2 , x 3 = A −1 b 3 ,..., x k = A −1 b kcan be obtained with one matrix inversion <strong>and</strong> k matrix multiplications. Once again,however, a more efficient method is to form the matrix[A|b 1 |b 2 | ··· |b k ] (1.13)inwhich the coefficient matrix A is “augmented” by all k <strong>of</strong> the matrices <strong>and</strong> then reduce(1.13) to reduced row-echelon form by Gauss-Jordan elimination. In this way we cansolve all k systems at once. This method has the added advantage that it applies evenwhen A is not invertible.Example 1.34 Solve the systems(a) 2x 1 −4x 2 = −10x 1 −3x 2 + x 4 = −4x 1 − x 3 +2x 4 = 43x 1 −4x 2 +3x 3 − x 4 = −11


Prepared by Dr.Archara Pacheenburawana (MA131 Sec 750001) 46Solution .........Properties <strong>of</strong> Invertible <strong>Matrices</strong>(b) 2x 1 −4x 2 = −8x 1 −3x 2 + x 4 = −2x 1 − x 3 +2x 4 = 93x 1 −4x 2 +3x 3 − x 4 = −15Up to now, to show that an n×n matrix A is invertible, it has been necessary to findan n×n matrix B such thatAB = I <strong>and</strong> BA = IThenext theorem shows thatif we produce ann×nmatrixB satisfying either condition,then the other condition holds automatically.Theorem 1.17 Let A be a square matrix.(a) If B is a square matrix satisfying BA = I, then B = A −1 .(b) If B is a square matrix satisfying AB = I, then B = A −1 .Theorem 1.18 (Equivalent Statements) If A is an n×n matrix, then the followingare equivalent.(a) A is invertible.(b) Ax = 0 has only the trivial solution.(c) The reduced row-echelon form <strong>of</strong> A is I n .(d) A is expressible as a product <strong>of</strong> elementary matrices.(e) Ax = b is consistent for every n×1 matrix b.(f) Ax = b has exactly one solution for every n×1 matrix b.Theorem 1.19 Let A <strong>and</strong> B be square matrices <strong>of</strong> the same size. If AB is invertible,then A <strong>and</strong> B must also be invertible.The following fundamental problem will occur frequently in various contexts.A Fundamental Problem: Let A be a fixed m×n matrix. Find allm×1 matrices b such that the system <strong>of</strong> equations Ax = b is consistent.If Ais aninvertible matrix, Theorem 1.16 completely solves this problem by assertingthat for every m × 1 matrix b, the linear system Ax = b has the unique solutionx = A −1 b. If A is not square, or if A is square but not invertible, then Theorem 1.16does not apply. In this case the matrix b must usually satisfy certain conditions in orderfor Ax = b to be consistent.


Prepared by Dr.Archara Pacheenburawana (MA131 Sec 750001) 47Example 1.35 What conditions must b 1 , b 2 , <strong>and</strong> b 3 satisfy in order for the system <strong>of</strong>equationsto be consistent?Solution .........x 1 +x 2 +2x 3 = b 1x 1 +x 3 = b 22x 1 +x 2 +3x 3 = b 3Example 1.36 What conditions must b 1 , b 2 , <strong>and</strong> b 3 satisfy in order for the system <strong>of</strong>equationsto be consistent?Solution .........x 1 −2x 2 + x 3 = b 12x 1 −5x 2 +2x 3 = b 23x 1 +2x 2 − x 3 = b 3Exercise 1.61. Solve the following system by inverting the coefficient matrix <strong>and</strong> using Theorem1.16.(a) x 1 + x 2 = 25x 1 +6x 2 = 9(c) 5x 1 +3x 2 +2x 3 = 43x 1 +3x 2 +2x 3 = 2x 2 + x 3 = 5(b) 2x 1 − x 2 = 86x 1 −5x 2 = 32(d) y + z = 63x−y + z = −7x+y −3z = −13(e) − x−2y −3z = 0w+ x+4y +4z = 7w +3x+7y +9z = 4−w −2x−4y −6z = 6(f) x 1 + 2x 2 −3x 3 = 82x 1 + 5x 2 −6x 3 = 17−x 1 − 2x 2 + x 3 −x 4 = −84x 1 +10x 2 −9x 3 +x 4 = 332. Solve the following general system by inverting the coefficient matrix <strong>and</strong> usingTheorem 1.16.x 1 +2x 2 +x 3 = b 1x 1 − x 2 +x 3 = b 2x 1 + x 2 = b 3Use the resulting formulas to find the solution if(a) b 1 = −1, b 2 = 3, b 3 = 4 (b) b 1 = 5, b 2 = 0, b 3 = 0(c) b 1 = −1, b 2 = −1, b 3 = 3


Prepared by Dr.Archara Pacheenburawana (MA131 Sec 750001) 483. Use the method <strong>of</strong> Example 1.34 to solve the systems in all part simultaneously.(a) x 1 −5x 2 = b 13x 1 +2x 2 = b 2i. b 1 = 1, b 2 = 4ii. b 1 = −2, b 2 = 5(b) x 1 −2x 2 = b 13x 1 + x 2 = b 2i. b 1 = −3, b 2 = 12ii. b 1 = −2, b 2 = 1iii. b 1 = −3, b 2 = 9(c) 4x 1 −7x 2 = b 1x 1 +2x 2 = b 2i. b 1 = 0, b 2 = 1ii. b 1 = −4, b 2 = 6iii. b 1 = −1, b 2 = 3iv. b 1 = −5, b 2 = 1(d) 2x 1 − x 2 + x 3 = b 1−2x 1 + x 2 −3x 3 = b 24x 1 −3x 2 + x 3 = b 3i. b 1 = 1, b 2 = −5, b 3 = 5ii. b 1 = 1, b 2 = −3, b 3 = 1iii. b 1 = 8, b 2 = −14, b 3 = 144. Solve the systems in both parts at the same time.(a) x 1 −2x 2 +2x 3 = −22x 1 −5x 2 + x 3 = 13x 1 −7x 2 +2x 3 = −1(b) x 1 −2x 2 +2x 3 = 12x 1 −5x 2 + x 3 = −13x 1 −7x 2 +2x 3 = 05. Find conditions that the b’s must satisfy for the system to be consistent.(a) 6x 1 −4x 2 = b 1 (b) x 1 −2x 2 +5x 3 = b 13x 1 −2x 2 = b 2 4x 1 −5x 2 +8x 3 = b 2−3x 1 +3x 2 −3x 3 = b 3(c) x 1 − x 2 +3x 3 +2x 4 = b 1−2x 1 + x 2 +5x 3 + x 4 = b 2−3x 1 +2x 2 +2x 3 − x 4 = b 34x 1 −3x 2 +3x 3 +3x 4 = b 46. Solve the following matrix equation for X.⎡ ⎤ ⎡ ⎤1 −1 1 2 −1 5 7 8⎣2 3 0 ⎦X = ⎣4 0 −3 0 1⎦0 2 −1 3 5 −7 2 17. In each part, determine whether the homogeneous system has a nontrivial solution(without using pencil <strong>and</strong> paper); then state whether the given matrix is invertible.


Prepared by Dr.Archara Pacheenburawana (MA131 Sec 750001) 49(a) 3x 1 −2x 2 + x 3 + x 4 = 04x 2 +5x 3 −2x 4 = 0x 3 −3x 4 = 02x 4 = 0(b) 4x 1 +3x 2 − x 3 +2x 4 = 03x 3 − x 4 = 0x 3 +5x 4 = 09x 4 = 0⎡ ⎤3 −2 1 1⎢0 4 5 −2⎥⎣0 0 1 −3⎦0 0 0 2⎡ ⎤4 3 −1 2⎢0 0 3 −1⎥⎣0 0 1 5 ⎦0 0 0 9Answer to Exercise 1.61. (a) x 1 = 3, x 2 = −1 (b) x 1 = 2, x 2 = −4(c) x 1 = 1, x 2 = −11, x 3 = 16 (d) x = −3, y = 2, z = 4(e) w = −6, x = 1, y = 10, z = −7(f) x 1 = 3, x 2 = 1, x 3 = −1, x 4 = 22. (a) x 1 = 163 , x 2 = − 4 3 , x 3 = − 11 3(b) x 1 = − 5 3 , x 2 = 5 3 , x 3 = 10 3(c) x 1 = 3,x 2 = 0,x 3 = −43. (a) i. x 1 = 2217 , x 2 = 117ii. x 1 = 2117 , x 2 = 1117(b) i. x 1 = −1, x 2 = 1 ii. x 1 = 2, x 2 = −5 iii. x 1 = 3, x 2 = 0(c) i. x 1 = 7 , x 15 2 = 4 15ii. x 1 = 34,15 2 = 2815iii. x 1 = 19,15 2 = 1315iv. x 1 = − 1, x 5 2 = 3 5(d) i. x 1 = −3, x 2 = −5, x 3 = 2 ii. x 1 = 0, x 2 = 0, x 3 = 1iii. x 1 = 2, x 2 = −1, x 3 = 34. (a) x 1 = −12−3t, x 2 = −5−t, x 3 = t(b) x 1 = 7−3t, x 2 = 3−t, x 3 = t5. (a) b 1 = 2b 2 (b) b 1 = b 2 +b 3 (c) b 1 = b 3 +b 4 , b 2 = 2b 3 +b 4⎡⎤11 12 −3 27 266. X = ⎣−6 −8 1 −18 −17⎦−15 −21 9 −38 −357. (a) Only the trivial solution x 1 = x 2 = x 3 = x 4 = 0; invertible(b) Infinitely many solutions; no invertible1.7 LU-DecompositionsWith Gaussian elimination <strong>and</strong> Gauss-Jordan elimination, a linear system is solved byoperating systematically on an augmented matrix. In this section we shall discuss adifferent organization <strong>of</strong> this approach, one based on factoring the coefficient matrix intoa product <strong>of</strong> lower <strong>and</strong> upper triangular matrices.


Prepared by Dr.Archara Pacheenburawana (MA131 Sec 750001) 50Solving <strong>Linear</strong> <strong>Systems</strong> by FactoringWe shall proceed in two stages. First, we shall show how a linear system Ax = b can besolved very easily once the coefficient matrix A is factored into a product <strong>of</strong> lower <strong>and</strong>upper triangular matrices. Second, we shall show how to construct such factorizations.If an n×n matrix A can be factored into a product <strong>of</strong> n×n matrices asA = LUwhere L is lower triangular <strong>and</strong> U is upper triangular, then the linear system Ax = bcan be solved as follows:Step 1. Rewrite the system Ax = b asStep 2. Define a new n×1 matrix y byLUx = b (1.14)Ux = y (1.15)Step 3. Use (1.15) to rewrite (1.14) as Ly = b <strong>and</strong> solve this system for y.Step 4. Substitute y in (1.15) <strong>and</strong> solve for x.The following example illustrates this procedure.Example 1.37 Given the factorization⎡ ⎤ ⎡ ⎤⎡2 1 3 2 0 0⎣ 4 1 7 ⎦ = ⎣ 4 −1 0 ⎦⎣−6 −2 −12 −6 1 −21 1 2320 1 −10 0 1use this result <strong>and</strong> the method described above to solve the system⎡ ⎤⎡⎤ ⎡ ⎤2 1 3 x 1 −1⎣ 4 1 7 ⎦⎣x 2⎦ = ⎣ 5 ⎦−6 −2 −12 x 3 −2Solution .........LU-DecompositionsWenowturntotheproblem<strong>of</strong>constructing suchfactorizations. Tomotivatethemethod,suppose that an n×n matrix A has been reduced to a row-echelon form U by Gaussianelimination—that is, by a certain sequence <strong>of</strong> elementary row operations. By Theorem1.12 each <strong>of</strong> these operations can be accomplished by multiplying on the left by anappropriate elementary matrix. Thus there are elementary matrices E 1 , E 2 , ..., E k suchthatE k···E 2 E 1 A = U (1.16)⎤⎦


Prepared by Dr.Archara Pacheenburawana (MA131 Sec 750001) 51By Theorem 1.13, E 1 , E 2 , ..., E k are invertible, so we can multiply both sides <strong>of</strong> Equation(1.16)on the left successively byE −1k ,...,E−1 2 ,E−1 1to obtainA = E1 −1 E−1 2 ···E −1k U (1.17)We can show that the matrix L defined byL = E −11 E−1 2 ···E −1k(1.18)is lower triangular provided that no row interchanges are used in reducing A to U.Substituting (1.18) into (1.17) yieldsA = LUwhich is a factorization <strong>of</strong> A into a product <strong>of</strong> a lower triangular matrix <strong>and</strong> an uppertriangular matrix.The following theorem summarizes the above result.Theorem 1.20 If A is a square matrix that can be reduced to a row-echelon form U byGaussian elimination without row interchange, then A can be factored as A = LU, whereL is a lower triangular matrix.Definition 1.16 A factorization <strong>of</strong> a square matrix A as A = LU, where L is lowertriangular <strong>and</strong> U is upper triangular, is called an LU-decomposition or triangulardecomposition <strong>of</strong> the matrix A.Example 1.38 Find an LU-decomposition <strong>of</strong>⎡2 1⎤3A = ⎣ 4 1 7 ⎦−6 −2 −12Solution To obtain an LU-decomposition, A = LU, we shall reduce A to a row-echelonform U using Gaussian elimination <strong>and</strong> then calculate L from (1.18). The steps are asfollows:Elementary MatrixReduction to Corresponding to Inverse <strong>of</strong> theRow-Echelon Form the Row Operation Elementary Matrix⎡2 1⎤3⎣ 4 1 7 ⎦−6 −2 −12⎡ ⎤ ⎡ ⎤10 0 2 0 02Step 1 E 1 = ⎣0 1 0⎦E1 −1 = ⎣0 1 0⎦0 0 1 0 0 1⎡⎣112324 1 7−6 −2 −12⎤⎦


Prepared by Dr.Archara Pacheenburawana (MA131 Sec 750001) 52Elementary MatrixReduction to Corresponding to Inverse <strong>of</strong> theRow-Echelon Form the Row⎡Operation⎤Elementary⎡Matrix⎤1 0 0 1 0 0Step 2 E 2 = ⎣−4 1 0⎦E2 −1 = ⎣4 1 0⎦0 0 1 0 0 1⎡⎣112320 −1 1−6 −2 −12⎡ ⎤1 0 0Step 3 E 3 = ⎣0 1 0⎦6 0 1⎡1⎣12320 −1 10 1 −3⎡ ⎤1 0 0Step 4 E 4 = ⎣0 −1 0⎦0 0 1⎡ ⎤⎣1 1 2320 1 −10 1 −3⎦⎡ ⎤1 0 0Step 5 E 5 = ⎣0 1 0⎦0 −1 1⎡⎣1 1 2320 1 −10 0 −2⎡1 0 0Step 6 E 6 = ⎣0 1 00 0 − 1 2⎡⎣1 1 2320 1 −10 0 1⎤⎦⎤⎦⎤⎦⎤⎦⎤⎦E −13 =⎡ ⎤1 0 0⎣ 0 1 0⎦−6 0 1⎡ ⎤1 0 0E4 −1 = ⎣0 −1 0⎦0 0 1⎡ ⎤1 0 0E5 −1 = ⎣0 1 0⎦0 1 1⎡ ⎤1 0 0E6 −1 = ⎣0 1 0 ⎦0 0 −2Thus⎡U = ⎣1 1 2320 1 −10 0 1⎤⎦


Prepared by Dr.Archara Pacheenburawana (MA131 Sec 750001) 53<strong>and</strong>, from (1.18),⎡ ⎤⎡⎤⎡⎤⎡⎤⎡⎤⎡⎤2 0 0 1 0 0 1 0 0 1 0 0 1 0 0 1 0 0L = ⎣0 1 0⎦⎣4 1 0⎦⎣0 1 0⎦⎣0 −1 0⎦⎣0 1 0⎦⎣0 1 0 ⎦0 0 1 0 0 1 −6 0 1 0 0 1 0 1 1 0 0 −2⎡ ⎤2 0 0= ⎣ 4 −1 0 ⎦−6 1 −2sois an LU-decomposition <strong>of</strong> A. ♣⎡ ⎤ ⎡ ⎤⎡2 1 3 2 0 0⎣ 4 1 7 ⎦ = ⎣ 4 −1 0 ⎦⎣−6 −2 −12 −6 1 −2Procedure for Finding LU-Decompositions1 1 2320 1 −10 0 1As this example shows, most <strong>of</strong> the work in constructing an LU-decomposition is expendedin the calculation <strong>of</strong> L. However, all this work can be eliminated by some carefulbookkeeping <strong>of</strong> the operations used to reduce A to U. Because we are assuming that norow interchanges are required to reduce A to U, there are only two types <strong>of</strong> operationsinvolved:• multiplying a row by a nonzero constant, <strong>and</strong>• adding a multiple <strong>of</strong> one row to another.The first operation is used to introduce the leading 1’s <strong>and</strong> the second to introduce zerosbelow the leading 1’s.In Example 1.38, the multiplier needed to introduce the leading 1’s in successive rowswere as follows:1for the first row2−1 for the second row− 1 for the third row2Note that in (1.19), the successive diagonal entries in L were precisely the reciprocals <strong>of</strong>these multipliers: ⎡ ⎤2 0 0L = ⎣ 4 −1 0 ⎦ (1.19)−6 1 −2Next, observe that to introduce zeros below the leading 1 in the first row, we used theoperationsadd −4 times the first row to the secondadd 6 times the first row to the third<strong>and</strong> to introduce the zero below the leading 1 in the second row, we used the operationadd −1 times the the second to the third⎤⎦


Prepared by Dr.Archara Pacheenburawana (MA131 Sec 750001) 54Now note in (1.20) that in each position below diagonal <strong>of</strong> L, the entry is the negative<strong>of</strong> the multiplier in the operation that introduced the zero in that position in U:⎡ ⎤2 0 0L = ⎣ 4 −1 0 ⎦ (1.20)−6 1 −2Therefore, we have the following procedure for constructing an LU-decomposition <strong>of</strong>a square matrix A provided that A can be reduced to row-echelon form without rowinterchanges.Step 1 Reduce A to a row-echelon form U by Gaussian elimination without row interchanges,keeping track <strong>of</strong> the multiplier used to introduce the leading 1’s.Step 2 In each position along the main diagonal <strong>of</strong> L, place the reciprocal <strong>of</strong> the multiplierthat introduced the leading 1 in that position <strong>of</strong> U.Step 3 In each position below the main diagonal <strong>of</strong> L, place the negative <strong>of</strong> the multiplierused to introduce the zero in that position in U.Example 1.39 Find an LU-decomposition <strong>of</strong>⎡ ⎤2 4 2A = ⎣1 5 2⎦4 −1 9Solution .........Example 1.40 Find an LU-decomposition <strong>of</strong>⎡ ⎤6 −2 0A = ⎣9 −1 1⎦3 7 5Solution .........Exercise 1.71. Use the method <strong>of</strong> Example 1.37 <strong>and</strong> the LU-decomposition[ ] [ ][ ]3 −6 3 0 1 −2=−2 5 −2 1 0 1to solve the system3x 1 −6x 2 = 0−2x 1 +5x 2 = 1


Prepared by Dr.Archara Pacheenburawana (MA131 Sec 750001) 552. Use the method <strong>of</strong> Example 1.37 <strong>and</strong> the LU-decomposition⎡ ⎤ ⎡ ⎤⎡⎤2 6 2 2 0 0 1 3 1⎣−3 −8 0⎦ = ⎣−3 1 0⎦⎣0 1 3⎦4 9 2 4 −3 7 0 0 1to solve the system2x 1 +6x 2 +2x 3 = 2−3x 1 −8x 2 = 24x 1 +9x 2 +2x 3 = 33. Find an LU-decomposition <strong>of</strong> the following coefficient matrix; then use the method<strong>of</strong> Example 1.37 to solve the system.⎡ ⎤⎡⎤ ⎡ ⎤[ ][ ] [ ] 2 −2 −2 x 2 8 x1 −21 −4(a) = (b) ⎣ 0 −2 2 ⎦⎣x −1 −1 x 2 −22⎦ = ⎣−2⎦−1 5 2 x 3 64. Let⎡ ⎤⎡⎤ ⎡ ⎤ ⎡ ⎤⎡⎤ ⎡ ⎤5 5 10 x 1 0 1 2 3 x 1 0(c) ⎣−8 −7 −9⎦⎣x 2⎦ = ⎣1⎦ (d) ⎣2 9 6 ⎦⎣x 2⎦ = ⎣10⎦0 4 26 x 3 4 3 2 10 x 3 7⎡ ⎤⎡⎤ ⎡ ⎤1 2 1 x 1 4(e) ⎣2 −1 −1⎦⎣x 2⎦ = ⎣1⎦1 1 3 x 3 0⎡ ⎤⎡⎤ ⎡ ⎤−1 0 1 0 x 1 5(g) ⎢ 2 3 −2 6⎥⎢x 2⎥⎣ 0 −1 2 0⎦⎣x 3⎦ = ⎢−1⎥⎣ 3 ⎦0 0 1 5 x 4 7⎡ ⎤⎡⎤ ⎡ ⎤1 −2 0 3 x 1 11(h) ⎢−2 3 1 −6⎥⎢x 2⎥⎣−1 4 −4 3 ⎦⎣x 3⎦ = ⎢−21⎥⎣−1⎦5 −8 4 0 x 4 23Find an LU-decomposition <strong>of</strong> A.⎡ ⎤⎡⎤ ⎡ ⎤1 −2 1 x 1 2(f) ⎣ 2 1 −1⎦⎣x 2⎦ = ⎣ 1 ⎦−3 1 −2 x 3 −5⎡ ⎤2 1 −1A = ⎣−2 −1 2 ⎦2 1 0Answer to Exercise 1.71. x 1 = 2, x 2 = 1 2. x 1 = 2, x 2 = −1, x 3 = 2


Prepared by Dr.Archara Pacheenburawana (MA131 Sec 750001) 563. (a) x 1 = 3, x 2 = −1 (b) x 1 = −1, x 2 = 1, x 3 = 0 (c) x 1 = −1, x 2 = 1, x 3 = 0(d) x 1 = −49, x 2 = 2, x 3 = 15 (e) x 1 = 1, x 2 = 2, x 3 = −1(f) x 1 = −49, x 2 = 0, x 3 = 1 (g) x 1 = −3, x 2 = 1, x 3 = 2, x 4 = 1(h) x 1 = 3, x 2 = −1, x 3 = 0, x 4 = 2⎡ ⎤⎡⎤2 0 0 1 1 − 1 2 24. A = LU = ⎣−2 1 0⎦⎣0 0 1 ⎦2 1 1 0 0 0


<strong>Chapter</strong> 2Determinants2.1 Determinants by C<strong>of</strong>actor ExpansionRecall from Theorem 1.6 that the 2×2 matrix[ ] a bA =c dis invertible if ad − bc ≠ 0. The expression ad − bc is called the determinant <strong>of</strong> thematrix A <strong>and</strong> is denoted by the symbol det(A) or |A|. With this notation, the formulafor A −1 given in Theorem 1.6 isA −1 =1det(A)[ ] d −b−c aOne <strong>of</strong> the goals <strong>of</strong> this chapter is to obtain analog <strong>of</strong> this formula to square matrix <strong>of</strong>higher order. This will require that we extend the concept <strong>of</strong> a determinant to squarematrices <strong>of</strong> all orders.Minors <strong>and</strong> C<strong>of</strong>actorsThere are several ways in which we might proceed. The approach in this section isa recursive approach. It defined the determinant <strong>of</strong> an n × n matrix in terms <strong>of</strong> thedeterminants <strong>of</strong> certain (n−1)×(n−1) matrices. The (n−1)×(n−1) matrices thatwill appear in this definition are submatrices <strong>of</strong> the original matrix. These submatricesare given a special name:Definition 2.1 If A is a square matrix, then the minor <strong>of</strong> entry a ij is denoted by M ij<strong>and</strong> is defined to be the determinant <strong>of</strong> the submatrix that remains after the ith row <strong>and</strong>jth column are deleted from A. The number (−1) i+j M ij is denoted by C ij <strong>and</strong> is calledthe c<strong>of</strong>actor <strong>of</strong> entry a ij .Example 2.1 LetFine the minor <strong>and</strong> c<strong>of</strong>actor <strong>of</strong> a 11 <strong>and</strong> a 32 .⎡ ⎤3 1 −4A = ⎣2 5 6 ⎦1 4 857


Prepared by Dr.Archara Pacheenburawana (MA131 Sec 750001) 58Solution .........Note that the c<strong>of</strong>actor <strong>and</strong> the minor <strong>of</strong> an element a ij differ only in sign; that is,C ij = ±M ij . A quick way to determine whether to use + or − is to use the fact that thesign relating C ij <strong>and</strong> M ij is in the ith row <strong>and</strong> jth column <strong>of</strong> the “checkerboard” array⎡ ⎤+ − + − + ···− + − + − ···+ − + − + ···⎢⎣− + − + − ··· ⎥⎦. . . . .For example, C 11 = M 11 , C 21 = −M 21 , C 13 = M 13 , C 32 = −M 32 , <strong>and</strong> so on.C<strong>of</strong>actor ExpansionsThe definition <strong>of</strong> a 3×3 determinant in terms <strong>of</strong> minors <strong>and</strong> c<strong>of</strong>actors isdet(A) = a 11 M 11 +a 12 (−M 12 )+a 13 M 13= a 11 C 11 +a 12 C 12 +a 13 C 13 (2.1)Equation (2.1) show that the determinant <strong>of</strong> A can be computed by multiplying theentries in the first row <strong>of</strong> A by their corresponding c<strong>of</strong>actors <strong>and</strong> adding the resultingproducts. More generally, we define the determinant <strong>of</strong> an n×n matrix to bedet(A) = a 11 C 11 +a 12 C 12 +···+a 1n C 1nThis method <strong>of</strong> evaluating det(A) is called c<strong>of</strong>actor expansion along the first row <strong>of</strong>A.⎡ ⎤3 1 0Example 2.2 Let A = ⎣−2 −4 3 ⎦. Evaluate det(A) by c<strong>of</strong>actor expansion along5 4 −2the first row <strong>of</strong> A.Solution .........If A is a 3×3 matrix, then its determinant is∣ a 11 a 12 a 13∣∣∣∣∣det(A) =a 21 a 22 a 23∣a 31 a 32 a 33∣ ∣ ∣ ∣ ∣ ∣ ∣∣∣ a= a 22 a 23∣∣∣∣∣∣ a11 −a 21 a 23∣∣∣∣∣∣ aa 32 a 12 +a 21 a 22∣∣∣33 a 31 a 1333 a 31 a 32= a 11 (a 22 a 33 −a 23 a 32 )−a 12 (a 21 a 33 −a 23 a 31 )+a 13 (a 21 a 32 −a 22 a 31 ) (2.2)= a 11 a 22 a 33 +a 12 a 23 a 31 +a 13 a 21 a 32 −a 13 a 22 a 31−a 12 a 21 a 33 −a 11 a 23 a 32 (2.3)


Prepared by Dr.Archara Pacheenburawana (MA131 Sec 750001) 59By rearranging the terms in (2.3) in various ways, it is possible to obtain other formulaslike (2.2). There should be no trouble checking that all <strong>of</strong> the following are correct:det(A) = a 11 C 11 +a 12 C 12 +a 13 C 13= a 11 C 11 +a 21 C 21 +a 31 C 31= a 21 C 21 +a 22 C 22 +a 23 C 23= a 12 C 12 +a 22 C 22 +a 32 C 32= a 31 C 31 +a 32 C 32 +a 33 C 33= a 13 C 13 +a 23 C 23 +a 33 C 33 (2.4)Note that in each equation, the entries <strong>and</strong> c<strong>of</strong>actors all come from the same row orcolumn. These equations are called the c<strong>of</strong>actor expansions <strong>of</strong> det(A).The results we have just given for 3×3 matrices from a special case <strong>of</strong> the followinggeneral theorem.Theorem 2.1 Expansions by C<strong>of</strong>actorsThe determinant <strong>of</strong> an n×n matrix A can be computed by multiplying the entries in anyrow (or column) by their c<strong>of</strong>actors <strong>and</strong> adding the resulting products; that is, for each1 ≤ i ≤ n <strong>and</strong> 1 ≤ j ≤ n,<strong>and</strong>det(A) = a 1j C 1j +a 2j C 2j +···+a nj C nj(c<strong>of</strong>actor expansion along the jth column)det(A) = a i1 C i1 +a i2 C i2 +···+a in C in(c<strong>of</strong>actor expansion along the ith row)Note that we may choose any row or any column.Example 2.3 Let A be the matrix in Example 2.2. Evaluate det(A) by c<strong>of</strong>actor expansionalong the first column <strong>of</strong> A.Solution .........RemarkIngeneral, thebest strategyforevaluating adeterminant byc<strong>of</strong>actorexpansionis to exp<strong>and</strong> along a row or column having the largest number <strong>of</strong> zeros.Example 2.4 Let⎡ ⎤1 0 0 −1A = ⎢3 1 2 2⎥⎣1 0 −2 1 ⎦2 0 0 1Find det(A) by a c<strong>of</strong>actor expansion along a row or column <strong>of</strong> your choice.Solution .........


Prepared by Dr.Archara Pacheenburawana (MA131 Sec 750001) 60Adjoint <strong>of</strong> a MatrixDefinition 2.2 If A is any n×n matrix <strong>and</strong> C ij is the c<strong>of</strong>actor <strong>of</strong> a ij , then the matrix⎡ ⎤C 11 C 12 ··· C 1nC 21 C 22 ··· C 2n⎢ ⎥⎣ . . . ⎦C n1 C n2 ··· C nnis call matrix <strong>of</strong> c<strong>of</strong>actors from A. The transpose <strong>of</strong> this matrix is called the adjointmatrix <strong>of</strong> A <strong>and</strong> is denoted by adj(A).Example 2.5 Find adj(A) when⎡ ⎤3 2 −1A = ⎣1 6 3 ⎦.2 −4 0Solution .........Theorem 2.2 Inverse <strong>of</strong> a Matrix Using Its AdjointIf A is an invertible matrix, thenA −1 =1adj(A) (2.5)det(A)Example 2.6 Use (2.5) to find the inverse <strong>of</strong> the matrix A in Example 2.5.Solution .........Theorem 2.3 If A is an n×n triangular matrix ( upper triangular, lower triangular, ordiagonal ) , then det(A) is the product <strong>of</strong> the entries on the main diagonal <strong>of</strong> the matrix;that is, det(A) = a 11 a 22···a nn .Example 2.7 Find det(A) when⎡ ⎤2 7 −3 8 30 −3 7 5 1A =⎢0 0 6 7 6⎥⎣0 0 0 9 8⎦ .0 0 0 0 4Solution .........Cramer’s RuleThe next theorem provides a formula for the solution <strong>of</strong> certain linear systems <strong>of</strong> nequations in n unknowns. This formula, known as Cramer’s rule.


Prepared by Dr.Archara Pacheenburawana (MA131 Sec 750001) 61Theorem 2.4 If Ax = b is a system <strong>of</strong> n linear equations in n unknowns such thatdet(A) ≠ 0, then the system has a unique solution. This solution isx 1 = det(A 1)det(A) , x 2 = det(A 2)det(A) ,..., x n = det(A n)det(A)where A j is the matrix obtained by replacing the entries in the jth column <strong>of</strong> A by theentries in the matrix ⎡ ⎤b 1b 2b = ⎢ ⎥⎣ . ⎦b nExample 2.8 Use Cramer’s rule to solveSolution .........x 1 +2x 3 = 6−3x 1 +4x 2 +6x 3 = 30−x 1 −2x 2 +3x 3 = 8Exercise 2.11. Let(a) Find all the minors <strong>of</strong> A.⎡ ⎤1 −2 3A = ⎣ 6 7 −1⎦−3 1 4(b) Find all the c<strong>of</strong>actors.2. Evaluate the determinant <strong>of</strong> the matrix in Exercise1 by a c<strong>of</strong>actor expansion along(a) the first row (b) the first column (c) the second row(d) the second column (e) the third row (f) the third column3. For the matrix in Exercise1, find(a) adj(A)(b) A −1 using its adjoint4. Evaluate det(A) by a c<strong>of</strong>actor expansion along a row or column <strong>of</strong> your choice.⎡3 3⎤1⎡ ⎤k +1 k −1 7(a) A = ⎣1 0 −4⎦ (b) A = ⎣ 2 k −3 4⎦1 −3 55 k +1 k5. Find A −1 using its adjoint.⎡ ⎤⎡ ⎤2 5 52 −3 5(a) A = ⎣−1 −1 0⎦ (b) A = ⎣0 1 −3⎦2 4 30 0 2


Prepared by Dr.Archara Pacheenburawana (MA131 Sec 750001) 626. Let⎡ ⎤1 3 1 1A = ⎢2 5 2 2⎥⎣1 3 8 9⎦1 3 2 2(a) Find A −1 using its adjoint.(b) Find A −1 using the method <strong>of</strong> Example 1.30 in Section 1.5.(c) Which method involves less computation?7. Show that the matrix⎡ ⎤cosθ sinθ 0A = ⎣−sinθ cosθ 0⎦0 0 1is invertible for all values <strong>of</strong> θ; then find A −1 using its adjoint.[ ]A11 A8. (a) If A = 12is an “upper triangular” block matrix, where A0 A 11 <strong>and</strong> A 2222are square matrices, then det(A) = det(A 11 )det(A 22 ). Use this result to evaluatedet(A) for⎡ ⎤2 −1 2 5 64 3 −1 3 4A =⎢ 0 0 1 3 5⎥⎣ 0 0 −2 6 2 ⎦0 0 3 5 2(b) Verify your answer in part(a) by using c<strong>of</strong>actor expansion to evaluate det(A).9. Solve by Cramer’s rule, where it applies.(a) 7x 1 −2x 2 = 33x 1 +x 2 = 5(b) x−4y +z = 64x−y +2z = −12x+2y −3z = −20(c) 3x 1 −x 2 +x 3 = 4−x 1 +7x 2 −2x 3 = 12x 1 +6x 2 −x 3 = 510. Solve the system4x+y +z +w = 63x+7y −z +w = 17x+3y −5z +8w = −3x+y +z +2w = 3by (a) Cramer’s rule(b) Guass-Jordan elimination.


Prepared by Dr.Archara Pacheenburawana (MA131 Sec 750001) 63Answer to Exercise 2.11. (a) M 11 = 29, M 12 = 21, M 13 = 27, M 21 = −11, M 22 = 13,M 23 = −5, M 31 = −19, M 32 = −19, M 33 = 19(b) C 11 = 29, C 12 = −21, C 13 = 27, C 21 = 11, C 22 = 13, C 23 = 5,C 31 = −19, C 32 = 19, C 33 = 192. 152⎡⎡ ⎤29 11 −193. (a) adj(A) = ⎣−21 13 19 ⎦ (b) A −1 = ⎢⎣27 5 1929152− 211522715211− 19152 1521315251524. (a) −66 (b) k 3 −8k 2 −10k +95⎡ ⎤ ⎡ ⎤1 33 −5 −5 15. (a) A −1 ⎣−3 4 5 ⎦ (b) A −1 2 2= ⎣0 1 3 ⎦22 −2 −3 0 0 1 2⎡ ⎤−4 3 0 −1 ⎡ ⎤6. A −1 = ⎢ 2 −1 0 0cosθ −sinθ 0⎥⎣−7 0 −1 8 ⎦ 7. A−1 = ⎣sinθ cosθ 0⎦0 0 16 0 1 −78. det(A) = 10×(−108) = −10809. (a) x 1 = 1, x 2 = 2 (b) x = − 144,y = 55 −61,z = 4655 11(c) Cramer’s rule do not apply.10. x = 1, y = 0, z = 2, w = 01915219152⎤⎥⎦2.2 Evaluating Determinants by Row ReductionIn this section we shall show that the determinant <strong>of</strong> a square matrix can be evaluatedby reducing the matrix to row-echelon form. This method is important since it is themost computationally efficient way to find the determinant <strong>of</strong> a general matrix.We begin with a fundamental theorem that will lead us to an efficient procedure forevaluating the determinant <strong>of</strong> a matrix <strong>of</strong> any order n.Theorem 2.5 Let A be a square matrix. If A has a row <strong>of</strong> zeros or a column <strong>of</strong> zeros,then det(A) = 0.Theorem 2.6 Let A be a square matrix. Then det(A) = det(A T ).


Prepared by Dr.Archara Pacheenburawana (MA131 Sec 750001) 64Theorem 2.7 Let A be a square matrix.1. If B is the matrix that results when a single row or single column <strong>of</strong> A is multipliedby a scalar k, then det(B) = kdet(A).2. If B is the matrix that results when two rows or two columns <strong>of</strong> A are interchanged,then det(B) = −det(A).3. If B is the matrix that results when a multiple <strong>of</strong> one row <strong>of</strong> A is added to anotherrow or when a multiple <strong>of</strong> one column is added to another column, then det(B) =det(A).The following table illustrates Theorem 2.7 for 3×3 determinants.Relationship∣ ∣ ka 11 ka 12 ka 13∣∣∣∣∣a 11 a 12 a 13∣∣∣∣∣a 21 a 22 a 23 = ka 21 a 22 a 23∣ a 31 a 32 a 33∣a 31 a 32 a 33det(B) = kdet(A)∣ ∣ a 21 a 22 a 23∣∣∣∣∣a 11 a 12 a 13∣∣∣∣∣a 11 a 12 a 13 = −a 21 a 22 a 23∣a 31 a 32 a 33∣a 31 a 32 a 33OperationThe first row <strong>of</strong> Ais multiplied by k.The first <strong>and</strong>second rows <strong>of</strong> Aare interchanged.det(B) = −det(A)∣ ∣ a 11 +ka 12 a 12 +ka 22 a 13 +ka 23∣∣∣∣∣a 11 a 12 a 13∣∣∣∣∣a 21 a 22 a 23 =a 21 a 22 a 23∣ a 31 a 32 a 33∣a 31 a 32 a 33A multiple <strong>of</strong> thesecond row <strong>of</strong> Ais added to thefirst row.Elementary <strong>Matrices</strong>det(B) = det(A)If we let A = I n in Theorem 2.7, then the matrix B is an elementary matrix, <strong>and</strong> thetheorem yields the following result about determinants <strong>of</strong> elementary matrices.Theorem 2.8 Let E be an n×n elementary matrix.1. If E results from multiplying a row <strong>of</strong> I n by k, then det(E) = k.2. If E results from interchanging two rows <strong>of</strong> I n , then det(E) = −1.3. If E results from adding a multiple <strong>of</strong> one row <strong>of</strong> I n to another, then det(E) = 1.


Prepared by Dr.Archara Pacheenburawana (MA131 Sec 750001) 65Example 2.9 Find the determinant <strong>of</strong> the following matrices.⎡ ⎤1 0 0 0(a) A = ⎢0 3 0 0⎥⎣0 0 1 0⎦0 0 0 1⎡ ⎤0 0 0 1(b) B = ⎢0 3 0 0⎥⎣0 0 1 0⎦1 0 0 0⎡ ⎤1 0 0 7(c) C = ⎢0 1 0 0⎥⎣0 0 1 0⎦0 0 0 1<strong>Matrices</strong> with Proportional Rows or ColumnsTheorem 2.9 If A is a square matrix with two proportional rows or two proportionalcolumns, then det(A) = 0.Each <strong>of</strong> the following matrices has two proportional rows or columns; thus, each hasa determinant <strong>of</strong> zero.⎡ ⎤⎡ ⎤[ ]3 −1 4 −51 −2 7−1 4, ⎣−4 8 5⎦,⎢ 6 −2 5 2⎥−2 8 ⎣ 5 8 1 4 ⎦2 −4 3−9 3 −12 15Evaluating Determinants by Row ReductionWe shall now give a method for evaluating determinants that involves substantially lesscomputation than the c<strong>of</strong>actor expansion method. The idea <strong>of</strong> this method is to reducethe given matrix to upper triangular from by elementary row operations, then computethe determinant <strong>of</strong> the upper triangular matrix (an easy computation), <strong>and</strong> then relatethat determinant to that <strong>of</strong> the original matrix. Here is an example:Example 2.10 Evaluate det(A) where⎡ ⎤0 1 5A = ⎣3 −6 9⎦2 6 1Solution .........Example 2.11 Compute the determinant <strong>of</strong>⎡ ⎤1 2 0 7A = ⎢0 7 6 3⎥⎣0 0 3 1 ⎦3 6 0 −5


Prepared by Dr.Archara Pacheenburawana (MA131 Sec 750001) 66SolutionExample 2.12 Evaluate det(A) where⎡ ⎤1 2 0 −2A = ⎢0 0 2 −1⎥⎣0 −1 1 0 ⎦1 3 4 1Solution .........C<strong>of</strong>actor expansion <strong>and</strong>row operationscan sometimes be used in combination to providean effective method for evaluating determinants. The following example illustratesthis idea.Example 2.13 Evaluate det(A) where⎡ ⎤3 5 −2 6A = ⎢1 2 −1 1⎥⎣2 4 1 5⎦3 7 5 3Solution .........Example 2.14 Evaluate det(A) where⎡ ⎤2 1 −3 1A = ⎢−3 −2 0 2⎥⎣ 2 1 0 −1⎦1 0 1 2Solution .........Example 2.15 Evaluate det(A) where⎡Solution .........⎢A = ⎣1 303 42−1 3 5 21− 3 58 4 4Exercise 2.2⎤⎥⎦1. Evaluate the following determinants.3 −17 4(a)0 5 1∣0 0 −2∣−2 1 3(c)1 −7 4∣−2 1 3∣√ 2 0 0 0(b)−8 2 0 07 0 −1 0∣ 9 5 6 1∣1 −2 3(d)2 −4 6∣5 −8 1∣


Prepared by Dr.Archara Pacheenburawana (MA131 Sec 750001) 672. Evaluatethedeterminant <strong>of</strong>thegivenmatrixbyreducingthematrixtorow-echelonform.⎡ ⎤⎡ ⎤3 6 91 −3 0(a) ⎣ 0 0 −2⎦(b) ⎣−2 4 1⎦−2 1 55 −2 2⎡ ⎤⎡(c) ⎢⎣1 −2 3 15 −9 6 3−1 2 −6 −22 8 6 1⎤⎥⎦a b c3. Given thatd e f= −6, find∣g h i∣ d e f(a)g h i∣a b c∣a+g b+h c+i(c)d e f∣ g h i ∣1 3 1 5 3−2 −7 0 −4 2(d)⎢ 0 0 1 0 1⎥⎣ 0 0 2 1 1⎦0 0 0 1 13a 3b 3c(b)−d −e −f∣4g 4h 4i ∣−3a −3b −3c(d)d e f∣g −4d h−4e i−4f∣4. Repeat Exercise 2 using a combination <strong>of</strong> row reduction <strong>and</strong> c<strong>of</strong>actor expansion,as in Example 2.13.5. Solve the equation ∣ ∣∣∣∣∣ x 5 70 x+1 60 0 2x−1∣ = 0Answer to Exercise 2.21. (a) −30 (b) −2 (c) 0 (d) 0 2. (a) 30 (b) −17 (c) 39 (d) −23. (a) −6 (b) 72 (c) −6 (d) 18 5. x = 0,−1, 1 22.3 Properties <strong>of</strong> the Determinant FunctionBasic Properties <strong>of</strong> DeterminantsSuppose that A <strong>and</strong> B are n×n matrices <strong>and</strong> k is any scalar. We begin by consideringpossible relationships between det(A), det(B), <strong>and</strong>det(kA), det(A+B), <strong>and</strong> det(AB)


Prepared by Dr.Archara Pacheenburawana (MA131 Sec 750001) 68Since a common factor <strong>of</strong> any row <strong>of</strong> a matrix can be moved though the det sign, <strong>and</strong>since each <strong>of</strong> the n rows in kA has a common factor <strong>of</strong> k, we obtaindet(kA) = k n det(A) (2.6)For example, ∣ ∣ ∣ ∣ ∣∣∣∣∣ ka 11 ka 12 ka 13∣∣∣∣∣∣∣∣∣∣ a 11 a 12 a 13∣∣∣∣∣ka 21 ka 22 ka 23 = k 3 a 21 a 22 a 23ka 31 ka 32 ka 33 a 31 a 32 a 33Unfortunately, no simple relationship exists among det(A), det(B), <strong>and</strong> det(A+B).In particular, we emphasize that det(A+B) will usually not be equal to det(A)+det(B).For example, ifA =[ ] 1 2, B =2 5[ ] 3 1, A+B =1 3then det(A) = 1, det(B) = 8, <strong>and</strong> det(A+B) = 23; thusdet(A+B) ≠ det(A)+det(B)[ ] 4 3,3 8But if we consider two 2×2 matrices that differ only in the second row:[ ] [ ]a11 aA = 12 a11 a<strong>and</strong> B = 12,a 21 a 22 b 21 b 22then we haveThusdet(A)+det(B) = (a 11 a 22 −a 12 a 21 )+(a 11 b 22 −a 12 b 21 )= a 11 (a 22 +b 22 )−a 12 (a 21 +b 21 )[ ]a= det 11 a 12a 21 +b 21 a 22 +b 22[ ] [ ] [ ]a11 adet 12 a11 a+det 12 a= det 11 a 12a 21 a 22 b 21 b 22 a 21 +b 21 a 22 +b 22This is a special case <strong>of</strong> the following theorem.Theorem 2.10 Let A,B, <strong>and</strong> C be n × n matrices that differ only in the single row,say the rth, <strong>and</strong> assume that the rth row <strong>of</strong> C can be obtained by adding correspondingentries in the rth row <strong>of</strong> A <strong>and</strong> B. Thendet(C) = det(A)+det(B)For example,⎡1 7 5⎤ ⎡ ⎤1 7 5⎡ ⎤1 7 5det⎣2 0 3 ⎦ = det⎣2 0 3⎦+det⎣2 0 3 ⎦1+0 4+1 7+(−1) 1 4 7 0 1 −1


Prepared by Dr.Archara Pacheenburawana (MA131 Sec 750001) 69Determinant <strong>of</strong> a Matrix ProductCorollary 2.1 If B is an n×n matrix <strong>and</strong> E is an n×nelementary matrix, thenDeterminant Test for Invertibilitydet(EB) = det(E)det(B)Theorem 2.11 A square matrix A is invertible if <strong>and</strong> only if det(A) ≠ 0.⎡ ⎤1 2 3Example 2.16 Determine whether the matrix A = ⎣1 0 1⎦ is invertible?2 4 6Solution .........Theorem 2.12 If A <strong>and</strong> B are square matrices <strong>of</strong> the same size, thenExample 2.17 Letdet(AB) = det(A)det(B)A =Show that det(AB) = det(A)det(B).Solution .........Theorem 2.13 If A is invertible, then[ ] 3 1, B =2 1det(A −1 ) =[ ] −1 35 81det(A) .We conclude this section by merging Theorem 2.11 with the list to produce thefollowing theorem that relates all the major topics we have studied thus far.Theorem 2.14 (Equivalent Statements) If A is an n×n matrix, then the followingstatements are equivalent.(a) A is invertible.(b) Ax = 0 has only the trivial solution.(c) The reduced row-echelon form <strong>of</strong> A is I n .(d) A can be expressed as a product <strong>of</strong> elementary matrices.(e) Ax = b is consistent for every n×1 matrix b.(f) Ax = b has exactly one solution for every n×1 matrix b.(g) det(A) ≠ 0.


Prepared by Dr.Archara Pacheenburawana (MA131 Sec 750001) 701. Verify that det(kA) = k n det(A) for(a) A =Exerise 2.3⎡ ⎤[ ]2 −1 3−1 2; k = 2 (b) A = ⎣3 2 1⎦ ; k = −23 41 4 52. Verify that det(AB) = det(A)det(B) for⎡ ⎤ ⎡ ⎤2 1 0 1 −1 3A = ⎣3 4 0⎦ <strong>and</strong> B = ⎣7 1 2⎦0 0 2 5 0 1Is det(A+B) = det(A)+det(B)?3. By inspection, explain why det(A) = 0.⎡−2 8 1 4A = ⎢⎣3 2 5 11 10 6 54 −6 4 −34. Use Theorem 2.11 to determine which <strong>of</strong> the following matrices are invertible.5. Let⎡ ⎤1 0 −1(a) ⎣9 −1 4 ⎦8 9 −1⎡√ √ ⎤ 2 − 7 0(c) ⎣3 √ 2 −3 √ 7 0⎦5 −9 0Assuming that det(A) = −7, find⎤⎥⎦⎡ ⎤4 2 8(b) ⎣−2 1 −4⎦3 1 6⎡ ⎤−3 0 1(d) ⎣ 5 0 6⎦8 0 3⎡ ⎤a b cA = ⎣d e f⎦g h i(a) det(3A) (b) det(A −1 ) (c) det(2A −1 )⎡ ⎤a g d(d) det(2A) −1 (e) det⎣b h e⎦c i f6. Without directly evaluating, show that x = 0 <strong>and</strong> x = 2 satisfyx 2 x 22 1 1∣0 0 −5∣ = 0


Prepared by Dr.Archara Pacheenburawana (MA131 Sec 750001) 717. For which value(s) <strong>of</strong> k does A fail to be invertible?(a) A =[ ] k −3 −2−2 k −2⎡ ⎤1 2 4(b) A = ⎣3 1 6⎦k 3 28. Indicate whether the statement is always true or sometimes false. Justify eachanswer by giving a logical argument or a counterexample.(a) det(2A) = 2det(A)(b) |A 2 | = |A| 2(c) det(I +A) = 1+det(A)(d) if det(A) = 0 then A is not expressible as a product <strong>of</strong> elementary matrices.(e) If the reduced row-echelon form <strong>of</strong> A has a row <strong>of</strong> zeros, then det(A) = 0.(f) There is no square matrix A such that det(AA T ) = −1.(g) If det(A) = 0, then the homogeneous system Ax = 0 has infinitely manysolutions.Answer to Exercise 2.31. (a) det(2A) = −40 = 2 2 det(A) (b) det(−2A) = −448 = (−2) 3 det(A)2. det(AB) = −170 = 10·(−17) = det(A)det(B)3. The second <strong>and</strong> forth columns <strong>of</strong> A are proportional.4. (a) Invertible (b) Not invertible (c) Not invertible (d) Not invertible5. (a) −63 (b) − 1 7(c) − 8 7(d) − 156(e) 76. If x = 0, the first <strong>and</strong> third rows are proportional.If x = 2, the first <strong>and</strong> second rows are proportional.7. (a) k = 5±√ 172(b) k = −18. (a) False (b) True (c) False (d) True (e) True (f) True (g) True


<strong>Chapter</strong> 3Vector Spaces3.1 Real Vector SpacesVector Space AxiomsDefinition 3.1 Let V be an arbitrary nonempty on which the operations <strong>of</strong> additions<strong>and</strong> scalar multiplication are defined. If the following axioms are satisfied by all objectsu, v, w in V <strong>and</strong> all scalars k <strong>and</strong> m, then we call V a vector space <strong>and</strong> we call theobjects in V vectors.1. If u <strong>and</strong> v are objects in V, then u+v is in V.2. u+v = v+u3. u+(v+w) = (u+v)+w4. There is an object 0 in V, called a zero vector for V, such that 0+u = u+0 = ufor all u in V.5. For each u in V, there is an object −u in V, called a negative <strong>of</strong> u, such thatu+(−u) = (−u)+u = 0.6. If k is any scalar <strong>and</strong> u is any object in V, then ku is in V.7. k(u+v) = ku+kv8. (k +m)u = ku+mu9. k(mu) = (km)u10. 1u = uRemark Vector spaces in which the scalars are complex numbers are called complexvector spaces, <strong>and</strong> those in which the scalars must be real are called real vectorspaces. In this chapter, all <strong>of</strong> our scalar will be real numbers.72


Prepared by Dr.Archara Pacheenburawana (MA131 Sec 750001) 73Examples <strong>of</strong> Vector SpacesExample 3.1 R n Is a Vector SpaceLet u = (u 1 ,u 2 ,...,u n ) <strong>and</strong> v = (v 1 ,v 2 ,...,v n ) are two vectors in R n .The sum u+v is defined byu+v = (u 1 +v 1 ,u 2 +v 2 ,...,u n +v n )<strong>and</strong> if k is any scalar, the scalar multiple ku is defined byku = (ku 1 ,ku 2 ,...,ku n )The operations <strong>of</strong> addition <strong>and</strong> scalar multiplication in the above are called the st<strong>and</strong>ardoperations on R n .We can show that the set V = R n with these st<strong>and</strong>ard operations is a vector space.♣The three most important special cases <strong>of</strong> R n are R (the real numbers), R 2 (thevectors in the plane), <strong>and</strong> R 3 (the vectors in 3-space).Example 3.2 A Vector Space <strong>of</strong> 2×2 <strong>Matrices</strong>Show that the set V <strong>of</strong> all 2 ×2 matrices with real entries is a vector space if additionis defined to be matrix addition <strong>and</strong> scalar multiplication is defined to be matrix scalarmultiplication.Solution It convenient to verify the axioms in the following order: 1, 6, 2, 3, 7, 8, 9, 4,5, <strong>and</strong> 10. Let [ ] [ ]u11 uu = 12 v11 v<strong>and</strong> v = 12u 21 u 22 v 21 v 22To prove Axiom 1, we must show that u+v is an object in V; that is, we must showthat u+v is a 2×2 matrix.[ ] [ ] [ ]u11 uu+v = 12 v11 v+ 12 u11 +v= 11 u 12 +v 12u 21 u 22 v 21 v 22 u 21 +v 21 u 22 +v 22Similarly, Axiom 6 holds because for any real scalar number k, we have[ ] [ ]u11 uku = k 12 ku11 ku= 12u 21 u 22 ku 21 ku 22so ku is a 2×2 matrix <strong>and</strong> consequently is an object in V.Axiom 2 follows from Theorem 1.2(a) since[ ] [ ] [ ] [ ]u11 uu+v = 12 v11 v+ 12 v11 v= 12 u11 u+ 12= v+uu 21 u 22 v 21 v 22 v 21 v 22 u 21 u 22Similarly, Axiom 3 follows from part (b) <strong>of</strong> that theorem; <strong>and</strong> Axioms 7, 8, <strong>and</strong> 9 followfrom parts (f), (h), <strong>and</strong> (j), respectively.


Prepared by Dr.Archara Pacheenburawana (MA131 Sec 750001) 74To prove Axiom 4, we must find an object 0 in V such that 0+u = u+0 = u forall u in V. This can be done by defining 0 to be[ ]0 00 =0 0With this definition,0+u =[ ] [ ] [ ]0 0 u11 u+ 12 u11 u= 12= u0 0 u 21 u 22 u 21 u 22<strong>and</strong> similarly u+0 = u. To prove Axiom 5, we must show that each object u in V hasa negative −u such that u+(−u) = 0 <strong>and</strong> (−u)+u = 0. This can be done by definingthe negative <strong>of</strong> u to be [ ]−u11 −u−u = 12−u 21 −u 22With this definition,u+(−u) =[ ] [ ]u11 u 12 −u11 −u+ 12=u 21 u 22 −u 21 −u 22[ ]0 0= 00 0<strong>and</strong> similarly (−u)+u = 0. Finally, Axiom 10 is a simple computation:[ ] [ ]u11 u1u = 1 12 u11 u= 12= u ♣u 21 u 22 u 21 u 22Example 3.2 is a special case <strong>of</strong> a vector space V <strong>of</strong> all m × n matrices with realentries, together with the operations <strong>of</strong> matrix addition <strong>and</strong> scalar multiplication. Weshall denote this vector space by the symbol M m×n .Example 3.3 A Vector Space <strong>of</strong> Real-Valued FunctionsLet V be the set <strong>of</strong> real-value functions defined on the entire real line (−∞,∞). Iff = f(x) <strong>and</strong> g = g(x) are two such functions <strong>and</strong> k is any real number, define the sumfunction f +g <strong>and</strong> the scalar multiple kf, respectively, by(f +g)(x) = f(x)+g(x) <strong>and</strong> (kf)(x) = kf(x)In the exercise we shall ask you to show that V is a vector space with respect to theseoperations. This vector space is denoted by F(−∞,∞). ♣


Prepared by Dr.Archara Pacheenburawana (MA131 Sec 750001) 75Example 3.4 The Vector Space P nLet P n denote the set <strong>of</strong> all polynomials <strong>of</strong> degree less than or equal n. If p <strong>and</strong> q are twopolynomials in P n <strong>and</strong> k is any real number, define the sum p+q <strong>and</strong> the scalar multiplekp, respectively, by(p+q)(x) = p(x)+q(x) <strong>and</strong> (kp)(x) = kp(x)for all real number x. In this case, the zero vector is the zero polynomial,z(x) = 0x n +0x n−1 +0x n−2 +···+0x+0It is easily verified that all the vector space axioms holds. thus P n , with the st<strong>and</strong>ardaddition <strong>and</strong> scalar multiplication <strong>of</strong> functions, is a vector space. ♣Example 3.5 The Zero Vector SpaceLet V consist <strong>of</strong> a single object, which we denote by 0, <strong>and</strong> define0+0 = 0 <strong>and</strong> k0 = 0for all scalar k. It is easy to check that all the vector space axioms are satisfied. We callthis the zero vector space.Example 3.6 A Set That Is Not a Vector SpaceLet V = R 2 <strong>and</strong> define addition <strong>and</strong> scalar multiplication operations as follows: If u =(u 1 ,u 2 ) <strong>and</strong> v = (v 1 ,v 2 ), then define<strong>and</strong> if k is any real number, then defineu+v = (u 1 +v 1 ,u 2 +v 2 )ku = (ku 1 ,0)For example,if u = (2,4),v = (−3,5), <strong>and</strong> k = 7, thenu+v = (2+(−3),4+5) = (−1,8)ku = 7u = (7·2,0) = (14,0)In the exercises we will ask you to show that the first nine vector space axioms aresatisfied; however, there are values <strong>of</strong> u for which Axiom 10 fails to hold. For example,if u = (u 1 ,u 2 ) is such that u 2 ≠ 0, then1u = 1(u 1 ,u 2 ) = (1·u 1 ,0) = (u 1 ,0) ≠ uThus V is not a vector space with the stated operations. ♣


Prepared by Dr.Archara Pacheenburawana (MA131 Sec 750001) 76Some Properties <strong>of</strong> VectorsTheorem 3.1 Let V be a vector space, u a vector in V, <strong>and</strong> k a scalar; then:(a) 0u = 0(b) k0 = 0(c) (−1)u = −u(d) If ku = 0, then k = 0 or u = 0.Exercise 3.11. A set <strong>of</strong> objects is given, together with operations <strong>of</strong> addition <strong>and</strong> scalar multiplication.Determine which sets are vector spaces under the given operations. Forthose that are not vector spaces, list all axioms that fail to hold.(a) The set <strong>of</strong> all triples <strong>of</strong> real numbers (x,y,z) with the operations(x,y,z)+(x ′ ,y ′ ,z ′ ) = (x+x ′ ,y +y ′ ,z +z ′ ) <strong>and</strong> k(x,y,z) = (kx,y,z)(b) The set <strong>of</strong> all pairs <strong>of</strong> real numbers (x,y) with the operations(x,y)+(x ′ ,y ′ ) = (x+x ′ ,y +y ′ ) <strong>and</strong> k(x,y) = (2kx,2ky)(c) The set <strong>of</strong> all pairs <strong>of</strong> real numbers <strong>of</strong> the form (x,0) with the st<strong>and</strong>ardoperations on R 2 .(d) The set <strong>of</strong> all n-tuples <strong>of</strong> real numbers <strong>of</strong> the form (x,x,...,x) with thest<strong>and</strong>ard operations on R n .(e) The set <strong>of</strong> 2×2 matrices <strong>of</strong> the form[ ] a 11 bwith the st<strong>and</strong>ard matrix addition <strong>and</strong> scalar multiplication.(f) The set <strong>of</strong> all real-valued functions f defined everywhere on the real line <strong>and</strong>such that f(1) = 0, with the operations defined in Example 3.3.(g) The set <strong>of</strong> all pairs <strong>of</strong> real numbers <strong>of</strong> the form (1,x) with the operations(1,y)+(1,y ′ ) = (1,y +y ′ ) <strong>and</strong> k(1,y) = (1,ky)(h) The set <strong>of</strong> polynomials <strong>of</strong> the form a+bx with the operations(a 0 +a 1 x)+(b 0 +b 1 x) = (a 0 +b 0 )+(a 1 +b 1 )x <strong>and</strong> k(a 0 +a 1 x) = (ka 0 )+(ka 1 )x2. Complete the unfinished details <strong>of</strong> Example 3.3.


Prepared by Dr.Archara Pacheenburawana (MA131 Sec 750001) 773. Show thatthefirst ninevector spaceaxiomsaresatisfied ifV = R 2 hastheaddition<strong>and</strong> scalar multiplication operations defined in Example 3.6.1. (a) Not a vector space. Axiom 8 fails.Answer to Exercise 3.1(b) Not a vector space. Axiom 9 <strong>and</strong> 10 fail.(c) The set is a vector space under the given operations.(d) The set is a vector space under the given operations.(e) Not a vector space. Axiom 1,4,5, <strong>and</strong> 6 fail.(f) The set is a vector space under the given operations.(g) The set is a vector space under the given operations.(h) The set is a vector space under the given operations.3.2 SubspacesGiven a vector space V, it is <strong>of</strong>ten possible to form another vector space by taking asubset S <strong>of</strong> V <strong>and</strong> using the operations <strong>of</strong> V.Definition 3.2 A subset S <strong>of</strong> a vector space V is called subspaces <strong>of</strong> V if S is itself avector space under the addition <strong>and</strong> scalar multiplication defined on V.In general, one must verify the ten vector space axioms to show that a set S withaddition <strong>and</strong> scalar multiplication forms a vector space. However, if S is part <strong>of</strong> thelarger set V that is already known to be a vector space, then certain axioms need notbe verified for S because they are “inherited” from V. For example, there is no need tocheck that u+v = v + u (Axiom 2) for S because this holds for all vectors in V <strong>and</strong>consequently for all vectors in S. Other axioms inherited by S from V are 3,7,8,9, <strong>and</strong>10. Thus to show that the set S is a subspace <strong>of</strong> a vector space V, we need only verifyAxioms 1,4,5, <strong>and</strong> 6. The following theorem shows that even Axioms 4 <strong>and</strong> 5 can beomitted.Theorem 3.2 If S is a nonempty subset <strong>of</strong> a vector space V, then S is a subspace <strong>of</strong> Vif <strong>and</strong> only if the following conditions hold.(a) If u <strong>and</strong> v are vectors in S, then u+v is in S.(b) If k is any scalar <strong>and</strong> u in any vector in S, then ku is in S.Remark1. A set S <strong>of</strong> one or more vectors form a vector space V is said to be closedunder addition if condition (a) in Theorem 3.2 holds <strong>and</strong> closed under scalarmultiplication if condition (b) holds. Thus Theorem 3.2 states that S is a subspace <strong>of</strong>V if <strong>and</strong> only if S is closed under addition <strong>and</strong> closed under scalar multiplication.2. Every nonempty vector space V has at least two subspaces: V itself is a subspace,<strong>and</strong> the set {0} consisting <strong>of</strong> just the zero vector in V is a subspace called the zerosubspace.


Prepared by Dr.Archara Pacheenburawana (MA131 Sec 750001) 78Example 3.7 Let S = {(x 1 ,x 2 ,x 3 ) ∈ R 3 |x 1 = x 2 }. Show that S is a subspace <strong>of</strong> R 3 .Solution .........Example 3.8 Let S = {A ∈ M 2×2 |a 12 = −a 21 }. Show that S is a subspace <strong>of</strong> M 2×2 .Solution .........Example 3.9 Let S be the set <strong>of</strong> all polynomials <strong>of</strong> degree less than n with the propertyp(0) = 0. Show that S is a subspace <strong>of</strong> P n .Solution .........Example 3.10 Let C n [a,b] be the set <strong>of</strong> all functions f that have a continuous nthderivative on [a,b]. We can verify that C n [a,b] is a subspace <strong>of</strong> C[a,b]. ♣Remark C[a,b] denote the set <strong>of</strong> all real-value functions that defined <strong>and</strong> continuous onthe closed interval [a,b].Example 3.11 Let S be the set <strong>of</strong> all functions f in C 2 [a,b] such thatf ′′ (x)+f(x) = 0for all x in [a,b]. Show that S is a subspace <strong>of</strong> C 2 [a,b]Solution .........Example 3.12 A Subset <strong>of</strong> R 2 That Is Not a SubspaceLet W be the set <strong>of</strong> all points (x,y) in R 2 such that x ≥ 0 <strong>and</strong> y ≥ 0. The set Wis not a subspace <strong>of</strong> R 2 since it is not closed under scalar multiplication. For example,u = (1,1) lies in W, but its negative (−1)u = −u = (−1,−1) does not. ♣{[ ] ∣x ∣∣∣Example 3.13 Let W = x is a real number}. Determine whether W is a sub-1space <strong>of</strong> M 2×1 .Solution .........Solution Spaces <strong>of</strong> Homogeneous <strong>Systems</strong>If Ax = b is a system <strong>of</strong> linear equations, then each vector x that satisfies this equationis called solution vector <strong>of</strong> the system. The following theorem shows that the solutionvectors <strong>of</strong> a homogeneous linear system form a vector space, which we shall call solutionspace <strong>of</strong> the system.Theorem 3.3 If Ax = 0 is a homogeneous linear system <strong>of</strong> m equations in n unknowns,then the set <strong>of</strong> solution vectors, denoted by N(A), is a subspace <strong>of</strong> R n .Note N(A) = {x ∈ R n |Ax = 0}


Prepared by Dr.Archara Pacheenburawana (MA131 Sec 750001) 79Example 3.14 Determine N(A) ifSolution .........A =[ ]1 1 1 02 1 0 1Definition 3.3 A vector w is called a linear combination <strong>of</strong> the vectors v 1 ,v 2 ,...,v rif it can be expressed in the formwhere k 1 ,k 2 ,...,k r are scalars.w = k 1 v 1 +k 2 v 2 +···+k r v rRemark If r = 1, then the equation in Definition 3.3 reduces to w = k 1 v 1 ; that is, wis a linear combination <strong>of</strong> a single vector v 1 if it is a scalar multiple <strong>of</strong> v 1 .Example 3.15 Every vector v = (a,b,c) in R 3 is expressible as a linear combination <strong>of</strong>the st<strong>and</strong>ard basis vectorssincei = (1,0,0), j = (0,1,0), k = (0,0,1)v = (a,b,c) = a(1,0,0)+b(0,1,0)+c(0,0,1) = ai+bj+ck ♣Example 3.16 Consider the vectors u = (1,2,−1) <strong>and</strong> v = (6,4,2) in R 3 . Show that(a) w = (9,2,7) is a linear combination <strong>of</strong> u <strong>and</strong> v <strong>and</strong>(b) w ′ = (4,−1,8) is not a linear combination <strong>of</strong> u <strong>and</strong> vSolution .........SpanningIf v 1 ,v 2 ,...,v r are vectors in vector space V, then generally some vectors in V may belinear combinations <strong>of</strong> v 1 ,v 2 ,...,v r <strong>and</strong> other may not. The following theorem showsthat if we construct a set W consisting <strong>of</strong> all those vectors that are expressible as linearcombinations <strong>of</strong> v 1 ,v 2 ,...,v r , then W forms a subspace <strong>of</strong> V.Theorem 3.4 If v 1 ,v 2 ,...,v r are vectors in a vector space V, then(a) The set W <strong>of</strong> all linear combinations <strong>of</strong> v 1 ,v 2 ,...,v r is a subspace <strong>of</strong> V.(b) W is the smallest subspace <strong>of</strong> V that contains v 1 ,v 2 ,...,v r in the sense that everyother subspace <strong>of</strong> V that contains v 1 , v 2 , ..., v r must contain W.Definition 3.4 If S = {v 1 ,v 2 ,...,v r } is the set <strong>of</strong> vectors in a vector space V, thenthe subspace W <strong>of</strong> V consisting <strong>of</strong> all linear combinations <strong>of</strong> the vector in S is called thespace spanned by v 1 , v 2 , ..., v r , <strong>and</strong> we say that the vectors v 1 , v 2 , ..., v r span W.To indicate that W is the space spanned by the vectors in the set S = {v 1 ,v 2 ,...,v r },we writeW = Span(S) or W = Span{v 1 ,v 2 ,...,v r }


Prepared by Dr.Archara Pacheenburawana (MA131 Sec 750001) 80Example 3.17 If v 1 <strong>and</strong> v 2 are noncollinear vectors in R 3 with their initial points atthe origin, then Span{v 1 ,v 2 }, which consists <strong>of</strong> all linear combinations k 1 v 1 + k 2 v 2 , isthe plane determined by v 1 <strong>and</strong> v 2 .zSpan{v 1 ,v 2 }k 2 v 2v 2v 1k 1 v 1k 1 v 1 +k 2 v 2yxSimilarly, if v is a nonzero vector in R 2 or R 3 , then Span{v}, which is the set <strong>of</strong> allscalar multiples kv, is the line determined by v.zvkvSpan{v}yxExample 3.18 The polynomial 1, x, x 2 , ..., x n span the vector space P n defined inExample 3.4 since each polynomial p in P n can be written asp = a 0 +a 1 x+···+a n x nwhich is a linear combination <strong>of</strong> 1,x,x 2 ,...,x n . We can denote this by writingP n = Span{1,x,x 2 ,...,x n } ♣Example 3.19 Determine whether v 1 = (1,1,2), v 2 = (1,0,1), <strong>and</strong> v 3 = (2,1,3) spanthe vector space R 3 .Solution .........Example 3.20 Determine whether the vectors 1−x 2 , x+2, <strong>and</strong> x 2 span P 3 .Solution .........


Prepared by Dr.Archara Pacheenburawana (MA131 Sec 750001) 81Theorem 3.5 If S = {v 1 ,v 2 ,...,v r } <strong>and</strong> S ′ = {w 1 ,w 2 ,...,w k } are two set <strong>of</strong> vectorsin a vector space V, thenSpan{v 1 ,v 2 ,...,v r } = Span{w 1 ,w 2 ,...,w k }if <strong>and</strong> only if each vector in S is a linear combination <strong>of</strong> those in S ′ <strong>and</strong> each vector inS ′ is a linear combination <strong>of</strong> those in S.Exercise 3.21. Use Theorem 3.2 to determine which <strong>of</strong> the following sets form subspaces <strong>of</strong> R 2 .(a) {(x 1 ,x 2 )|x 1 +x 2 = 0} (b) {(x 1 ,x 2 )|x 1 x 2 = 0}(c) {(x 1 ,x 2 )|x 1 = 3x 2 } (d) {(x 1 ,x 2 )||x 1 | = |x 2 |}(e) {(x 1 ,x 2 )|x 2 1 = x2 2 }2. Use Theorem 3.2 to determine which <strong>of</strong> the following are subspaces <strong>of</strong> R 3 .(a) all vectors <strong>of</strong> the form (a,0,0)(b) all vectors <strong>of</strong> the form (a,1,1)(c) all vectors <strong>of</strong> the form (a,b,c), where b = a+c(d) all vectors <strong>of</strong> the form (a,b,c), where b = a+c+1(e) all vectors <strong>of</strong> the form (a,b,0)(f) all vectors <strong>of</strong> the form (a,b,c), where a = b = c3. Use Theorem 3.2 to determine which <strong>of</strong> the following are subspaces <strong>of</strong> P 3 .(a) all polynomials a 0 +a 1 x+a 2 x 2 +a 3 x 3 for which a 0 = 0(b) all polynomials a 0 +a 1 x+a 2 x 2 +a 3 x 3 for which a 0 +a 1 +a 2 +a 3 = 0(c) all polynomials a 0 +a 1 x+a 2 x 2 +a 3 x 3 for which a 0 , a 1 , a 2 , <strong>and</strong> a 3 are integers(d) all polynomials p(x) <strong>of</strong> the form a 0 +a 1 x+a 2 x 2 +a 3 x 3 such that p(0) = 04. Use Theorem 3.2 to determine which <strong>of</strong> the following sets form subspaces <strong>of</strong> M 2×2 .(a) The set <strong>of</strong> all 2×2 diagonal matrices(b) The set <strong>of</strong> all 2×2 triangular matrices(c) The set <strong>of</strong> all 2×2 lower triangular matrices(d) The set <strong>of</strong> all 2×2 matrices A such that a 12 = 1(e) The set <strong>of</strong> all 2×2 matrices B such that b 11 = 0(f) The set <strong>of</strong> all symmetric 2×2 matrices(g) The set <strong>of</strong> all singular 2×2 matrices5. Use Theorem 3.2 to determine which <strong>of</strong> the following are subspaces <strong>of</strong> M n×n


Prepared by Dr.Archara Pacheenburawana (MA131 Sec 750001) 82(a) all n×n matrices A such that tr(A) = 0(b) all n×n matrices A such that A T = −A(c) all n×n matrices A such that the linear system Ax = 0 has only the trivialsolution(d) all n×n matrices A such that AB = BA for a fixed n×n matrix B6. Use Theorem 3.2 to determine which <strong>of</strong> the following sets form subspaces <strong>of</strong>C[−1,1].(a) The set <strong>of</strong> function f in C[−1,1] such that f(−1) = f(1)(b) The set <strong>of</strong> odd functions in C[−1,1](c) The set <strong>of</strong> continuous nondecreasing functions on [−1,1](d) The set <strong>of</strong> function f in C[−1,1] such that f(−1) = 0 <strong>and</strong> f(1) = 0(e) The set <strong>of</strong> function f in C[−1,1] such that f(−1) = 0 or f(1) = 07. Determine the nullspace <strong>of</strong> each <strong>of</strong> the following matrices.[ ][ ]2 11 2 −3 −1(a) A =(b) A =3 2−2 −4 6 3⎡ ⎤ ⎡ ⎤1 3 −41 1 −1 2(c) A = ⎣ 2 −1 −1⎦ (d) A = ⎣ 2 2 −3 1 ⎦−1 −3 4−1 −1 0 −58. Which<strong>of</strong>thefollowingarelinearcombinations<strong>of</strong>u = (0,−2,2)<strong>and</strong>v = (1,3,−10)?(a) (2,2,2) (b) (3,1,5) (c) (0,4,5) (d) (0,0,0)9. Express the following as linear combinations <strong>of</strong> u = (2,1,4), v = (1,−1,3), <strong>and</strong>w = (3,2,5).(a) (−9,−7,−15) (b) (6,11,6) (c) (0,0,0) (d) (7,8,9)10. Express the following as linear combinations <strong>of</strong> p 1 = 2+x+4x 2 , p 2 = 1−x+3x 2 ,<strong>and</strong> p 3 = 3+2x+5x 2 .(a) −9−7x−15x 2(b) 6+11x+6x 2(c) 0(d) 7+8x+9x 211. In each part, determine whether the given vectors span R 3 .(a) v 1 = (2,2,2), v 2 = (0,0,3), v 3 = (0,1,1)(b) v 1 = (2,−1,3), v 2 = (4,1,2), v 3 = (8,−1,8)(c) v 1 = (3,1,4), v 2 = (2,−3,5), v 3 = (5,−2,9), v 4 = (1,4,−1)


Prepared by Dr.Archara Pacheenburawana (MA131 Sec 750001) 83(d) v 1 = (1,2,6), v 2 = (3,4,1), v 3 = (4,3,1), v 4 = (3,3,1)12. Which <strong>of</strong> the following are spanning sets for P 2 ? Justify your answer.(a) {1,x 2 ,x 2 −2}(b) {2,x 2 ,x,2x+3}(c) {x+2,x+1,x 2 −1} (d) {x+2,x 2 −1}13. Let f = cos 2 x <strong>and</strong> g = sin 2 x. Which <strong>of</strong> the following lie in the space spanned byf <strong>and</strong> g?(a) cos2x (b) 3+x 2 (c) 1 (d) sinx (e) 014. In M 2×2 letE 11 =[ ] 1 0, E0 0 12 =Show that E 11 ,E 12 ,E 21 ,E 22 span M 2×2 .[ ] 0 1, E0 0 21 =Answer to Exercise 3.2[ ] 0 0, E1 0 22 =[ ] 0 00 11. (a), (c) 2. (a), (c), (f) 3. (a), (b), (d) 4. (a), (c), (e), (f) 5.(a), (b), (d)6. (a), (b), (d)7. (a) {(0,0)} (b) Span{(−2,1,0,0),(3,0,1,0)}(c) Span{(1,1,1)} (d) Span{(−5,0,−3,1),(−1,1,0,0)}8. (a), (b), (d)9. (a) −2(2,1,4)+(1,−1,3)−2(3,2,5) (b) 4(2,1,4)−5(1,−1,3)+(3,2,5)(c) 0(2,1,4)+0(1,−1,3)+0(3,2,5) (d) 0(2,1,4)−2(1,−1,3)+3(3,2,5)10. (a) −2p 1 +p 2 −2p 3 (b) 4p 1 −5p 2 +p 3 (c) 0p 1 +0p 2 +0p 3(d) 0p 1 −2p 2 +3p 311. (a) The vectors span. (b) The vectors do not span.(c) The vectors do not span. (d) The vectors span.12. (b), (c) 13. (a), (c), (e)3.3 <strong>Linear</strong> IndependenceIn this section we look more closely at the structure <strong>of</strong> vector spaces. To begin with, werestrict ourselves to vector spaces that can be generated from a finite set <strong>of</strong> elements.Each vector in the vector space can be built up from the elements in this generatingset using only the operations <strong>of</strong> addition <strong>and</strong> scalar multiplication. This generating setis usually referred to as a spanning set. In particular, it is desirable to find a minimal


Prepared by Dr.Archara Pacheenburawana (MA131 Sec 750001) 84spanning set. By minimal we mean a spanning set with no unnecessary elements (i.e.all the elements in the set are needed in order to span the vector space). To see how t<strong>of</strong>ind a minimal spanning set, it is necessary to consider how the vectors in the collectiondepend on each other. Consequently, we introduce the concepts <strong>of</strong> linear dependence<strong>and</strong> linear independence. These simple concepts provide the key to underst<strong>and</strong>ing thestructure <strong>of</strong> vector spaces.Consider the following vectors in R 3 :⎡ ⎤ ⎡ ⎤ ⎡ ⎤1x 1 = ⎣−12−2 −1⎦, x 2 = ⎣ 3 ⎦, x 3 = ⎣ 3 ⎦1 8Let S be the subspace <strong>of</strong> R 3 spanned by x 1 , x 2 , x 3 . Actually, S can be represented interms <strong>of</strong> two vectors x 1 <strong>and</strong> x 2 , since the vector x 3 is already in the span <strong>of</strong> x 1 <strong>and</strong> x 2 .x 3 = 3x 1 +2x 2 (3.1)Any linear combination <strong>of</strong> x 1 , x 2 , x 3 can be reduced to a linear combination <strong>of</strong> x 1 <strong>and</strong>x 2 :Thusα 1 x 1 +α 2 x 2 +α 3 x 3 = α 1 x 1 +α 2 x 2 +α 3 (3x 1 +2x 2 )= (α 1 +3α 3 )x 1 +(α 2 +2α 3 )x 2S = Span{x 1 ,x 2 ,x 3 } = Span{x 1 ,x 2 }Equation (3.1) can be rewritten in the form3x 1 +2x 2 −1x 3 = 0 (3.2)Since the three coefficients in (3.2) are nonzero, we could solve for any vector in terms<strong>of</strong> the other two.It follows thatx 1 = − 2 3 x 2 + 1 3 x 3, x 2 = − 3 2 x 1 + 1 2 x 3, x 3 = 3x 1 +2x 2Span{x 1 ,x 2 ,x 3 } = Span{x 2 ,x 3 } = Span{x 1 ,x 3 } = Span{x 1 ,x 2 }Because <strong>of</strong> the dependency relation (3.2), the subspace S can be represented as the span<strong>of</strong> any two <strong>of</strong> the given vectors.On the other h<strong>and</strong>, no such dependency relationship exists between x 1 <strong>and</strong> x 2 . Indeed,if there were scalars k 1 <strong>and</strong> k 2 , not both 0, such thatk 1 x 1 +k 2 x 2 = 0 (3.3)then we could solve for one <strong>of</strong> the vectors in terms <strong>of</strong> the other.x 1 = − k 2k 1x 2 (k 1 ≠ 0) or x 2 = − k 1k 2x 1 (k 2 ≠ 0)


Prepared by Dr.Archara Pacheenburawana (MA131 Sec 750001) 85However, neither <strong>of</strong> the two vectors in question is a multiple <strong>of</strong> the other. Therefore,Span{x 1 } <strong>and</strong> Span{x 2 } are both proper subspaces <strong>of</strong> Span{x 1 ,x 2 }, <strong>and</strong> the only waythat (3.3) can hold is if k 1 = k 2 = 0.We can generalize this example by making the following observations.(I) If {v 1 ,v 2 ,...,v n } span a vector space V <strong>and</strong> one <strong>of</strong> these vectors can be writtenas a linear combination <strong>of</strong> the other n−1 vectors, then those n−1 vectors spanV.(II) Given n vectors v 1 ,v 2 ,...,v n , it is possible to write one <strong>of</strong> the vectors as a linearcombination <strong>of</strong> the other n−1 vectors if <strong>and</strong> only if there exist scalars k 1 ,k 2 ,...,k nnot all zero such thatk 1 v 1 +k 2 v 2 +···+k n v n = 0Definition 3.5 If S = {v 1 ,v 2 ,...,v r } is a nonempty set <strong>of</strong> vectors, then the vectorequationk 1 v 1 +k 2 v 2 +···+k r v r = 0has at least one solution, namelyk 1 = 0, k 2 = 0,..., k r = 0If this is the only solution, then S is called a linearly independent set. If there areother solutions, then S is called a linearly dependent set.It follows from (I) <strong>and</strong> (II) that, if {v 1 ,v 2 ,...,v r } is a minimal spanning set, thenv 1 ,v 2 ,...,v r are linearly independent. Conversely, if v 1 ,v 2 ,...,v r are linearly independent<strong>and</strong> span V, then {v 1 ,v 2 ,...,v r } is a minimal spanning set for V.Example 3.21 If v 1 = (2,−1,0,3), v 2 = (1,2,5,−1), <strong>and</strong> v 3 = (7,−1,5,8), then theset <strong>of</strong> vectors S = {v 1 ,v 2 ,v 3 } is linearly dependent, since 3v 1 +v 2 −v 3 = 0. ♣Example 3.22 The polynomialp 1 = 1−x, p 2 = 5+3x−2x 2 , p 3 = 1+3x−x 2form a linear dependent set in P 2 since 3p 1 −p 2 +2p 3 = 0. ♣Example 3.23 Consider the vectors i = (1,0,0), j = (0,1,0), <strong>and</strong> k = (0,0,1) in R 3 .In terms <strong>of</strong> components, the vector equationbecomesor, equivalently,k 1 i+k 2 j+k 3 k = 0k 1 (1,0,0)+k 2 (0,1,0)+k 3 (0,0,1) = (0,0,0)(k 1 ,k 2 ,k 2 ) = (0,0,0)This impliesthat k 1 = 0, k 2 = 0, <strong>and</strong> k 3 = 0, so the set S = {i,j,k} is linear independent.A similar argument can be used to show that the vectorse 1 = (1,0,0,...,0), e 2 = (0,1,0,...,0),..., e n = (0,0,0,...,1)form a linear independent set in R n . ♣


Prepared by Dr.Archara Pacheenburawana (MA131 Sec 750001) 86Example 3.24 Show that1, x, x 2 ,...,x nform a linearly independent set <strong>of</strong> vectors in P n .Solution .........Example 3.25 Determine whether the vectorsv 1 = (1,−2,3), v 2 = (5,6,−1), v 3 = (3,2,1)form a linear dependent set or a linear independent set.Solution .........Theorem 3.6 Let v 1 ,v 2 ,...,v n be n vectors in R n <strong>and</strong> let X = [ v 1 v 2 ··· v n]. Thevectors v 1 , v 2 , ..., v n will be linearly dependent if <strong>and</strong> only if X is singular.We can use Theorem 3.6 to test whether n vectors are linearly independent in R n .Simply form a matrix X whose columns are the vectors being tested. To determinewhether X is singular, calculate the value <strong>of</strong> det(X). If det(X) = 0, the vectors arelinearly dependent. If det(X) ≠ 0, the vectors are linearly independent.Example 3.26 Determine wether the vectorsare linearly dependent.Solution .........v 1 = (4,2,3), v 2 = (2,3,1), v 3 = (2,−5,3)To determine wether r vectors v 1 , v 2 , ..., v r in R n are linearly independent, we canrewrite the equationk 1 v 1 +k 2 v 2 +···+k r v r = 0as a linear system Xk = 0, where X = [ v 1 v 2 ··· v r]. If r ≠ n, then the matrix Xis not square, so we cannot use determinants to decide wether the vectors are linearlyindependent. The system is homogeneous, so it has the trivial solution k = 0. It willhave nontrivial solutions if <strong>and</strong> only if the row echelon forms <strong>of</strong> X involve free variables.If there are nontrivial solutions, then the vectors are linearly dependent. If there are n<strong>of</strong>ree variables, then k = 0 is the only solution, <strong>and</strong> hence the vectors must be linearlyindependent.Example 3.27 Givenv 1 = (1,−1,2,3), v 2 = (−2,3,1,−2), v 3 = (1,0,7,7)To determine if the vectors are linearly independent, we reduce the system Xk = 0 torow echelon form⎡ ⎤ ⎡ ⎤1 −2 1 0 1 −2 1 0⎢ ⎢⎣ −1 3 0 0⎥2 1 7 0 ⎦ −−→ ⎢ 0 1 1 0⎥⎣ 0 0 0 0 ⎦3 −2 7 0 0 0 0 0Since the echelon form involves a free variables k 3 , there are nontrivial solutions <strong>and</strong>hence the vectors must be linearly dependent. ♣


Prepared by Dr.Archara Pacheenburawana (MA131 Sec 750001) 87To determine whether a set <strong>of</strong> vectors is linear independent in R n , we must solve ahomogeneous linear system <strong>of</strong> equations. A similar situation holds for the vector spaceP n .Example 3.28 Determine whether the vectorsp 1 = x 2 −2x+3, p 2 = 2x 2 +x+8, p 3 = x 2 +8x+7form a linear dependent set or a linear independent set.Solution .........Next we consider a very important property <strong>of</strong> linearly independent vectors. <strong>Linear</strong>combinations <strong>of</strong> linearly independent vectors are unique. More precisely, we have thefollowing theorem.Theorem 3.7 Let v 1 , v 2 , ..., v n be vectors in a vector space V. A vector v inSpan{v 1 ,v 2 ,...,v n } can be written uniquely as a linear combination <strong>of</strong> v 1 , v 2 , ..., v nif <strong>and</strong> only if v 1 , v 2 , ..., v n are linearly independent.The following theorem gives two simple facts about linear independence that areimportant to know.Theorem 3.8(a) A finite set <strong>of</strong> vectors that contain the zero vector is linearly dependent.(b) A set with exactly two vectors is linear independent if <strong>and</strong> only if neither vector isa scalar multiple <strong>of</strong> the other.Geometric Interpretation <strong>of</strong> <strong>Linear</strong> Independence<strong>Linear</strong> independence has some useful geometric interpretations in R 2 <strong>and</strong> R 3 .If x <strong>and</strong> y are linearly dependent in R 2 , thenk 1 x+k 2 y = 0where k 1 <strong>and</strong> k 2 are not both 0. If, say, k 1 ≠ 0, we can writex = − k 2k 1yIf two vectors in R 2 are linearly dependent, one <strong>of</strong> the vectors can be written as a scalarmultiple <strong>of</strong> the other. Thus, if both vectors are placed at the origin, they will lie alongthe same line.


Prepared by Dr.Archara Pacheenburawana (MA131 Sec 750001) 88(x 1 ,x 2 )(y 1 ,y 2 )(x 1 ,x 2 )xy(y 1 ,y 2 )x <strong>and</strong> y linearly dependentx <strong>and</strong> y linearly independentIf⎡ ⎤ ⎡ ⎤x 1 y 1x = ⎣x 2⎦ <strong>and</strong> y = ⎣y 2⎦x 3 y 3are linearly independent in R 3 , then the two points (x 1 ,x 2 ,x 3 ) <strong>and</strong> (y 1 ,y 2 ,y 3 ) will not lieon the same line through the origin in 3-space. Since (0,0,0), (x 1 ,x 2 ,x 3 ), <strong>and</strong> (y 1 ,y 2 ,y 3 )are not collinear, they determine the plane. If (z 1 ,z 2 ,z 3 ) lies on this plane, the vectorz = (z 1 ,z 2 ,z 3 ) can be written as a linear combination <strong>of</strong> x <strong>and</strong> y, <strong>and</strong> hence x, y, <strong>and</strong> zare linearly dependent. If (z 1 ,z 2 ,z 3 ) does not lie on the plane, the three vectors will belinearly independent.yzzyxxx, y, <strong>and</strong> z linearly dependentx, y, <strong>and</strong> z linearly independentThe next theorem shows that a linearly independent set in R n can contain at mostn vectors.Theorem 3.9 Let S = {v 1 ,v 2 ,...,v n } be a set <strong>of</strong> vectors in R n . If r > n, then S islinearly dependent.Remark Theorem 3.9 tells us that a set in R 2 with more than two vectors is linearlydependent <strong>and</strong> a set in R 3 with more than three vectors is linearly dependent.<strong>Linear</strong> Independence <strong>of</strong> FunctionsIn Example 3.26 a determinant was used to test wether three vectors were linearly independentin R 3 . Determinants can also be used to help to decide if a set <strong>of</strong> n vectors islinearly independent in C (n−1) [a,b]. Indeed, let f 1 , f 2 , ..., f n be elements <strong>of</strong> C (n−1) [a,b].


Prepared by Dr.Archara Pacheenburawana (MA131 Sec 750001) 89If these vectors are linearly dependent, then there exist scalars c 1 , c 2 , ..., c n not all zerosuch thatc 1 f 1 (x)+c 2 f 2 (x)+···+c n f n (x) = 0 (3.4)for each x ∈ [a,b]. Taking the derivative with respect to x <strong>of</strong> both sides <strong>of</strong> yieldsc 1 f 1 (x)+c 2 f 2 (x)+···+c n f n (x) = 0If we continue taking derivatives <strong>of</strong> both sides, we end up with the systemc 1 f 1 (x) +c 2 f 2 (x) +···+c n f n (x) = 0c 1 f ′ 1(x) +c 2 f ′ 2(x) +···+c n f ′ n(x) = 0. . . .c 1 f (n−1)1 (x)+c 2 f (n−1)2 (x)+···+c n f n (n−1) (x) = 0For each fixed x ∈ [a,b], the matrix equation⎡⎤⎡⎤ ⎡ ⎤f 1 (x) f 2 (x) ··· f n (x) c 1 0f 1(x) ′ f 2(x) ′ ··· f n(x)′ c 2⎢⎥⎢⎥⎣ . . . ⎦⎣. ⎦ = ⎢ ⎥⎣0. ⎦f (n−1)1 (x) f (n−1)2 (x) ··· f n(n−1) (x) c n 0(3.5)will have the same nontrivial solution (c 1 ,c 2 ,...,c n ) T . Thus, if f 1 , f 2 , ..., f n are linearlydependent in C (n−1) [a,b], then, for fixed x ∈ [a,b], the coefficient matrix <strong>of</strong> system (3.5)is singular, its determinant is zero.Definition 3.6 Let f 1 , f 2 , ..., f n be functions in C (n−1) [a,b] <strong>and</strong> define the functionW(x) on [a,b] byf 1 (x) f 2 (x) ··· f n (x)f 1 ′W(x) =(x) f′ 2 (x) ··· f′ n (x). . .∣f (n−1)1 (x) f (n−1)2 (x) ··· f n(n−1)(x) ∣The function W(x) is called the Wronskian <strong>of</strong> f 1 , f 2 , ..., f n .Theorem 3.10 Let f 1 , f 2 , ..., f n be elements <strong>of</strong> C (n−1) [a,b]. If there exists a point x 0in [a,b] such that W(x 0 ) ≠ 0, then f 1 , f 2 , ..., f n are linearly independent.Example 3.29 Show that 1, e x , <strong>and</strong> e 2x are linearly independent in C(−∞,∞).Solution .........Example 3.30 Show that the vectors 1, x, x 2 , <strong>and</strong> x 3 are linearly independent in P 3 .Solution .........Example 3.31 Determine whether the functions x 2 <strong>and</strong> x|x| are linearly independentin C[−1,1].Solution .........


Prepared by Dr.Archara Pacheenburawana (MA131 Sec 750001) 90Exercise 3.31. Explain why the following are linearly dependent sets <strong>of</strong> vectors.(a) v 1 = (1,−2,4) <strong>and</strong> v 2 = (5,−10,−20) in R 3(b) v 1 = (3,−1), v 2 = (4,5), v 3 = (−4,7) in R 2(c) p 1 = 3−2x+x 2 <strong>and</strong> p 2 = 6−4x+2x 2 in P 2[ ] [ ]−3 4 3 −4(d) A = <strong>and</strong> B = in M2 0 −2 0 2×22. Determine whether the following vectors are linearly independent in R 2 .(a) (2,1),(3,2) (b) (2,3),(4,6)(c) (−2,1),(1,3),(2,4)(d) (−1,2),(1,−2),(2,−4)(e) (1,2),(−1,1)3. Determine whether the following vectors are linearly independent in R 3 .(a) (1,0,0),(0,1,1),(1,0,1) (b) (1,0,0),(0,1,1),(1,0,1),(1,2,3)(c) (2,1,−2),(3,2,−2),(2,2,0) (d) (2,1,−2),(−2,−1,2),(4,2,−4)(e) (1,1,3),(0,2,1)4. Determine whether the following vectors are linearly independent in R 4 .(a) (3,8,7,−3),(1,5,3,−1),(2,−1,2,6),(1,4,0,3)(b) (0,0,2,2),(3,3,0,0),(1,1,0,−1)(c) (0,3,−3,−6),(−2,0,0,−6),(0,−4,−2,−2),(0,−8,4,−4)(d) (3,0,−3,6),(0,2,3,1),(0,−2,−2,0),(−2,1,2,1)5. Determine whether the following vectors are linearly independent in M 2×2 .(a)(c)[ ] 1 0,1 1[ ]1 0,0 1[ ] 0 10 0[ ]0 1,0 0[ ]2 30 2(b)[ ] 1 0,0 1[ ] 0 1,0 0[ ] 0 01 06. Determine whether the following vectors are linearly independent in P 3 .(a) 1,x 2 ,x 2 −2(c) x+2,x+1,x 2 −1(b) 2,x 2 ,x,2x+3(d) x+2,x 2 −17. (a) Show that the vectors v 1 = (0,,3,1,−1),v 2 = (6,0,5,1), <strong>and</strong> v 3 = (4,−7,1,3)form a linearly dependent set in R 4(b) Express each vector as a linear combination <strong>of</strong> the other two.


Prepared by Dr.Archara Pacheenburawana (MA131 Sec 750001) 918. For which real values <strong>of</strong> λ do the following vectors form a linear dependent set inR 3 ?v 1 = (λ,− 1 2 ,−1 2 ), v 2 = (− 1 2 ,λ,−1 2 ), v 3 = (− 1 2 ,−1 2 ,λ)9. For what values <strong>of</strong> α will the two vectors cos(x+α) <strong>and</strong> sinx be linearly dependentin C[−π,π]?10. Under what conditions is a set with one vector linearly independent?11. Indicate whether each statement is always true or sometimes false. Justify youranswer by giving a logical argument or a counterexample.(a) If {v 1 ,v 2 } is a linearly dependent set, then each vector is a scalar multiple <strong>of</strong>the other.(b) If {v 1 ,v 2 ,v 3 } is a linearly independent set, then so is the set{kv 1 ,kv 2 ,kv 3 } for every nonzero scalar k.(c) The converse <strong>of</strong> Theorem 3.8(a) is also true.(d) If v 1 ,v 2 ,...,v n span R n , then they are linearly independent.(e) If v 1 ,v 2 ,...,v n span a vector space V, then there are linearly independent.(f) If v 1 ,v 2 ,...,v r are vectors in a vector space V <strong>and</strong>Span{v 1 ,v 2 ,...,v r } = Span{v 1 ,v 2 ,...,v r−1 }then v 1 ,v 2 ,...,v r are linearly dependent.1. (a) v 2 is a scalar multiple <strong>of</strong> v 1 .Answer to Exercise 3.3(b) The vectors are linear dependent by Theorem 3.9.(c) p 2 is a scalar multiple <strong>of</strong> p 1 .(d) B is a scalar multiple <strong>of</strong> A.2. (a), (e) 3. (a), (e) 4. (a), (b), (c), (d) 5. (a), (b) 6. (c), (d)7. (b) v 1 = 2 7 v 2 − 3 7 v 3, v 2 = 7 2 v 1 + 3 2 v 3, v 3 = − 7 3 v 1 + 2 3 v 28. λ = − 1 , λ = 1 9. When α is an odd multiple <strong>of</strong> π/2.210. If <strong>and</strong> only if the vector is not zero.11. (a) False (b) True (c) False (d) True (e) False (f) True

Hooray! Your file is uploaded and ready to be published.

Saved successfully!

Ooh no, something went wrong!